You are on page 1of 73

CHAPTER 5

MERCHANDISING OPERATIONS
SUMMARY OF QUESTION TYPES BY LEARNING OBJECTIVE, LEVEL OF
DIFFICULTY, BLOOM’S TAXONOMY, CPA CODES, AND AACSB CODES
Item LO LOD Bloom’s CPA AACSB Item LO LOD Bloom’s CPA AACSB Item LO LOD Bloom’s CPA AACSB
True-False Statements
1. 1 E K F AN 15. 2 M AP F AN 29. 4 E K F AN
2. 1 E K F AN 16. 3 M K F AN 30. 5 M K F AN
3. 1 E K F AN 17. 3 M C F AN 31. 5 E AP F AN
4. 1 E K F AN 18. 3 E C F AN 32. 5 E K F AN
5. 1 E C F AN 19. 3 M AP F AN 33. 5 E K F AN
6. 1 M K F AN 20. 3 M C F AN 34. 5 E K F AN
7. 1 M K F AN 21. 4 M K F AN 35. 5 M K F AN
F AN 4 K F AN *36 6 K F AN
8. 2 M C 22. E E
.
2 F AN 4 K F AN *37 6 C F AN
9. M K 23. E M
.
2 C F AN 4 K F AN *38 6 K F AN
10. E 24. E E
.
2 C F AN 4 K F AN *39 6 K F AN
11. E 25. E E
.
2 K F AN 4 K F AN *40 6 C F AN
12. E 26. M M
.
2 AP F AN 4 K F AN *41 6 C F AN
13. E 27. M M
.
14. 2 M C F AN 28. 4 M K F AN

LOD: E = Easy M = Medium


Bloom’s: AP = Application C = Comprehension K = Knowledge
CPA: F = Financial Reporting
AACSB: AN = Analytic

*This topic is dealt with in an Appendix to the chapter.

Copyright © 2017 John Wiley & Sons Canada, Ltd. Unauthorized copying, distribution, or transmission of this page is prohibited
Test
5 -Bank
2 for Financial Accounting: Tools for Business Decision-Making, Seventh Canadian Edition

SUMMARY OF QUESTION TYPES BY LEARNING OBJECTIVE, LEVEL OF


DIFFICULTY, BLOOM’S TAXONOMY, CPA CODES, AND AACSB CODES
(CONT’D)
LO Bloom’ CP AACS L LO Bloom’ CP AACS L LO Bloom’ CP
Item LO Item Item AACSB
D s A B O D s A B O D s A
Multiple Choice Questions
42. 1 M C F AN 79. 3 E K F AN 116. 4 E K F AN
43. 1 E K F AN 80. 3 E C F AN 117. 4 M K F AN
44. 1 E K F AN 81. 3 E C F AN 118. 4 E K F AN
45. 1 E K F AN 82. 3 E C F AN 119. 4 M AP F AN
46. 1 M C F AN 83. 3 M K F AN 120. 4 M AP F AN
47. 1 M C F AN 84. 3 E K F AN 121. 5 M AP F AN
48. 1 E K F AN 85. 3 E C F AN 122. 5 M AP F AN
49. 1 M K F AN 86. 3 M AP F AN 123. 5 M C F AN
50. 1 E K F AN 87. 3 E C F AN 124. 5 E C F AN
51. 1 E K F AN 88. 3 E C F AN 125. 5 M C F AN
52. 1 E K F AN 89. 3 M C F AN 126. 5 E AP F AN
53. 1 M K F AN 90. 3 E C F AN 127. 5 M AP F AN
54. 1 M K F AN 91. 3 M C F AN 128. 5 E AP F AN
55. 1 E C F AN 92. 3 E K F AN 129. 5 E K F AN
56. 1 E K F AN 93. 3 E C F AN 130. 5 E K F AN
57. 1 E C F AN 94. 3 M C F AN 131. 5 M AP F AN
58. 1 E C F AN 95. 3 E AP F AN 132. 5 M AP F AN
59. 1 M C F AN 96. 3 M AP F AN *133. 6 M C F AN
60. 1 E C F AN 97. 3 M AP F AN *134. 6 M C F AN
61. 1 E C F AN 98. 3 E C F AN *135. 6 E C F AN
62. 1,6 M C F AN 99. 3 E C F AN *136. 6 E C F AN
63. 1,6 E K F AN 100. 3 E C F AN *137. 6 E C F AN
64. 1,6 E K F AN 101. 3 E C F AN *138. 6 E C F AN
65. 2 M C F AN 102. 4 E C F AN *139. 6 E C F AN
66. 2 E K F AN 103. 4 E K F AN *140. 6 M C F AN
67. 2 M AP F AN 104. 4 E K F AN *141. 6 M C F AN
68. 2 E C F AN 105. 4 E C F AN *142. 6 E C F AN
69. 2 M C F AN 106. 4 E C F AN *143. 6 E C F AN
70. 2 M C F AN 107. 4 M C F AN *144. 6 E AP F AN
71. 2 E C F AN 108. 4 E K F AN *145. 6 M AP F AN
72. 2 M C F AN 109. 4 E K F AN *146. 6 M C F AN
73. 2 E AP F AN 110. 4 E K F AN *147. 6 M AP F AN
74. 2 E C F AN 111. 4 H C F AN *148. 6 M AP F AN
75. 2 E K F AN 112. 4 M C F AN *149. 6 M AP F AN
76. 2 H C F AN 113. 4 E K F AN *150. 6 M C F AN
77. 3 E C F AN 114. 4 E K F AN
78. 3 M K F AN 115. 4 M C F AN

LOD: E = Easy M = Medium H = Hard


Bloom’s: AP = Application C = Comprehension K = Knowledge
CPA: F = Financial Reporting
AACSB: AN = Analytic

Copyright © 2017 John Wiley & Sons Canada, Ltd. Unauthorized copying, distribution, or transmission of this page is prohibited
5-3 Merchandising Operations

*This topic is dealt with in an Appendix to the chapter.

Copyright © 2017 John Wiley & Sons Canada, Ltd. Unauthorized copying, distribution, or transmission of this page is prohibited
Test
5 -Bank
4 for Financial Accounting: Tools for Business Decision-Making, Seventh Canadian Edition

SUMMARY OF QUESTION TYPES BY LEARNING OBJECTIVE, LEVEL OF


DIFFICULTY, BLOOM’S TAXONOMY, CPA CODES, AND AACSB CODES
(CONT’D)
Bloom’ CP AACS LO Bloom’ CP AACS LO Bloom’ CP
Item LO LOD Item LO Item LO AACSB
s A B D s A B D s A
Exercises
2 AP F AN 4 AP F AN 5, AP F AN
151. H 162. E *173. H
6
152. 2 M AP F AN 163. 4 E C F AN *174. 6 H AP F AN
153. 2,3 E AP F AN 164. 4 M AP F AN *175. 6 M AP F AN
2,3 AP F AN 4, AP F AN 6 AP F AN
154. M 165. E *176. E
5
2,3 AP F AN 4, AP F AN 6 AP F AN
155. E 166. E *177. E
5
2,3 AP F AN 4, AP F AN 6 AP F AN
156. E 167. E *178. E
5
2,3 AP F AN 4, AP F AN 6 AP F AN
157. H 168. E *179. M
5
2,3 AP F AN 4, AP F AN 6 AP F AN
158. H 169. M *180. E
5
159. 2,3,6 E AP F AN 170. 5 M AP F AN *181. 6 M AP F AN
160. 3 M AP F AN 171. 5 M AP F AN *182. 6 E AP F AN
3 AP F AN 5, AP F AN
161. E *172. M
6
Matching
*183 1– E, K F AN
. 3,5,6 M
Short-Answer Essay
1 C F AN 4 F AN 4, F, AN,E
184. M 188 E C 192. M C
5 E
185. 1,2,6 M C F AN 189. 4 M C F AN 193. 5 E C F AN
186. 1,6 E K F AN 190. 4 E C F AN *194. 6 E C F AN
4 F AN 4 F, AN,C
187. E C 191. M C
E
CPA Questions
195. 2,3 M C F AN 197. 4 M AN F AN *199. 6 M C F AN
196. 4 M K F AN *198. 6 E K F AN

LOD: E = Easy M = Medium H = Hard


Bloom’s: AN = Analysis AP = Application C = Comprehension K = Knowledge
CPA: E = Professional and Ethical Behaviour F = Financial Reporting
AACSB: AN = Analytic C = Communication E = Ethics

*This topic is dealt with in an Appendix to the chapter.

Copyright © 2017 John Wiley & Sons Canada, Ltd. Unauthorized copying, distribution, or transmission of this page is prohibited
5-5 Merchandising Operations

SUMMARY OF LEARNING OBJECTIVES BY QUESTION TYPE


Item Type Item Type Item Type Item Type Item Type Item Type Item Type
Learning Objective 1
1. TF 6. TF 45. MC 50. MC 55. MC 60. MC 183. Ma
2. TF 7. TF 46. MC 51. MC 56. MC 61. MC 184. SAE
3. TF 42. MC 47. MC 52. MC 57. MC 62. MC 185. SAE
4. TF 43. MC 48. MC 53. MC 58. MC 63. MC 186. SAE
5. TF 44. MC 49. MC 54. MC 59. MC 64. MC
Learning Objective 2
8. TF 13. TF 67. MC 72. MC 151. Ex 156. Ex 185. SAE
9. TF 14. TF 68. MC 73. MC 152. Ex 157. Ex 195. CP
10. TF 15. TF 69. MC 74. MC 153. Ex 158. Ex
11. TF 65. MC 70. MC 75. MC 154. Ex 159. Ex
12. TF 66. MC 71. MC 76. MC 155. Ex 183. Ma
Learning Objective 3
16. TF 78. MC 84. MC 90. MC 96. MC 153. Ex 159. Ex
17. TF 79. MC 85. MC 91. MC 97. MC 154. Ex 160. Ex
18. TF 80. MC 86. MC 92. MC 98. MC 155. Ex 161. Ex
19. TF 81. MC 87. MC 93. MC 99. MC 156. Ex 183. Ma
20. TF 82. MC 88. MC 94. MC 100. MC 157. Ex 195. CP
77. MC 83. MC 89. MC 95. MC 101. MC 158. Ex
Learning Objective 4
21. TF 28. TF 107. MC 114. MC 162. Ex 169. Ex 192. SAE
22. TF 29. TF 108. MC 115. MC 163. Ex 187. SAE 196. CP
23. TF 102. MC 109. MC 116. MC 164. Ex 187. SAE 197. CP
24. TF 103. MC 110. MC 117. MC 165. Ex 188. SAE
25. TF 104. MC 111. MC 118. MC 166. Ex 189. SAE
26. TF 105. MC 112. MC 119. MC 167. Ex 190. SAE
27. TF 106. MC 113. MC 120. MC 168. Ex 191. SAE
Learning Objective 5
30. TF 35. TF 125. MC 130. MC 167. Ex 172. Ex
31. TF 121. MC 126. MC 131. MC 168. Ex 173. Ex
32. TF 122. MC 127. MC 132. MC 169. Ex 183. Ma
33. TF 123. MC 128. MC 165. Ex 170. Ex 192. SAE
34. TF 124. MC 129. MC 166. Ex 171. Ex 193. SAE
*Learning Objective 6
*36. TF *134. MC *141. MC *148. MC *175. Ex *182. Ex
*37. TF *135. MC *142. MC *149. MC *176. Ex *183. Ma
*38. TF *136. MC *143. MC *150. MC *177. Ex *185. SAE
*39. TF *137. MC *144. MC *159. Ex *178. Ex *186. SAE
*40. TF *138. MC *145. MC *172. Ex *179. Ex *194. SAE
*41. TF *139. MC *146. MC *173. Ex *180. Ex *198. CP
*133. MC *140. MC *147. MC *174. Ex *181. Ex *199. CP

Note: TF = True-False MC = Multiple Choice Ma = Matching


Ex = Exercise SAE = Short-Answer Essay CP = CPA Questions
Copyright © 2017 John Wiley & Sons Canada, Ltd. Unauthorized copying, distribution, or transmission of this page is prohibited
Test
5 -Bank
6 for Financial Accounting: Tools for Business Decision-Making, Seventh Canadian Edition

*This topic is dealt with in an Appendix to the chapter.

Copyright © 2017 John Wiley & Sons Canada, Ltd. Unauthorized copying, distribution, or transmission of this page is prohibited
5-7 Merchandising Operations

CHAPTER LEARNING OBJECTIVES

1. Identify the differences between service and merchandising companies. A service company performs
services. It has service or fee revenue and operating expenses. A merchandising company sells goods. It
has sales revenue, cost of goods sold, and gross profit in addition to operating expenses. Both types of
company may also report non-operating items and each would report income tax expense.

2. Prepare entries for purchases under a perpetual inventory system. The Inventory account is debited
for all purchases of merchandise and for freight costs if those costs are paid by the buyer (shipping terms
FOB shipping point). It is credited for purchase discounts, and purchase returns and allowances.

3. Prepare entries for sales under a perpetual inventory system. When inventory is sold, two entries are
required: (1) Cash or Accounts Receivable is debited and Sales is credited for the selling price of the
merchandise, and (2) Cost of Goods Sold is debited and Inventory is credited for the cost of inventory
items sold. Contra revenue accounts are used to record sales returns and allowances and sales discounts.
Two journal entries are also required for sales returns so that both the selling price and the cost of the
returned merchandise are recorded. Freight costs paid by the seller (shipping terms FOB destination) are
recorded as an operating expense.

4. Prepare a single-step and a multiple-step income statement. In a single-step income statement, all
data (except for income tax expense) are classified under two categories—revenues or expenses—and
income before income tax is determined in one step. Income tax expense is separated from the other
expenses and reported separately after income before income tax to determine net income (loss).
A multiple-step income statement shows several steps in determining profit. Step 1 deducts sales returns
and allowances and sales discounts from gross sales to determine net sales. Step 2 deducts the cost of
goods sold from net sales to determine gross profit. Step 3 deducts operating expenses (which can be
classified by nature or by function) from gross profit to determine income from operations. Step 4 adds or
deducts any non-operating items to determine income before income tax. Finally, step 5 deducts income
tax expense to determine net income (loss).

5. Calculate the gross profit margin and profit margin. The gross profit margin, calculated by dividing
gross profit by net sales, measures the gross profit earned for each dollar of sales. The profit margin,
calculated by dividing net income by net sales, measures the net income earned for each dollar of sales.
Both are measures of profitability that are closely watched by management and other interested parties.

*6. Prepare entries for purchases and sales under a periodic inventory system and calculate cost of
goods sold (Appendix 5A). The periodic inventory system differs from the perpetual inventory system in
that separate temporary accounts are used in the periodic system to record (1) purchases, (2) purchase
returns and allowances, (3) purchase discounts, and (4) freight costs that are paid by the buyer (shipping
terms FOB shipping point). The formula for cost of goods purchased is as follows: Purchases – purchase
returns and allowances – purchase discounts = net purchases; and net purchases + freight in = cost of
goods purchased.
Both systems use temporary accounts to record (1) sales, (2) sales returns and allowances, and (3) sales
discounts. However, in a periodic inventory system, only one journal entry is made to record a sale of
merchandise as the cost of goods sold is not recorded throughout the period. Instead, the cost of goods

Copyright © 2017 John Wiley & Sons Canada, Ltd. Unauthorized copying, distribution, or transmission of this page is prohibited
Test
5 -Bank
8 for Financial Accounting: Tools for Business Decision-Making, Seventh Canadian Edition

sold is determined at the end of the period.


To determine the cost of goods sold, first calculate the cost of goods purchased, as indicated above. Then,
calculate the cost of goods sold as follows: Beginning inventory + cost of goods purchased = cost of goods
available for sale; and cost of goods available for sale – ending inventory = cost of goods sold.
At the end of the period, the Inventory account is adjusted to reflect its proper balance as determined from
the inventory count results. The change in this account is allocated to the Cost of Goods Sold account as
are the balances in the Freight In and Purchases account and any related contra accounts.

Copyright © 2017 John Wiley & Sons Canada, Ltd. Unauthorized copying, distribution, or transmission of this page is prohibited
5-9 Merchandising Operations

TRUE-FALSE STATEMENTS

1. A physical inventory count should be done at least once a year regardless of whether a perpetual or periodic
inventory system is being used.

2. The operating cycle of a merchandising company is generally shorter than that of a service company.

3. Under a perpetual inventory system, the cost of goods sold is determined each time a sale occurs.

4. Inventory is usually the largest current asset for a merchandiser.

5. Cost of Goods Sold is considered an operating expense for a merchandising company.

6. Operating expenses are subtracted from revenue for a service company and from gross profit for a
merchandising company.

7. Cost of goods available for sale is considered an operating expense for a merchandising company.

8. When the terms of sale are FOB shipping point, the seller is responsible for any damages to the goods
during shipping.

9. Freight terms will specify the point at which ownership of the goods is transferred from the seller to the
buyer.

10. Freight costs incurred on incoming merchandise are an operating expense to the buyer.

11. The terms 2/10, n/30 mean that a 2% discount is allowed on payments made over 10 days but within the
credit period.

12. Discounts taken for early payment of an invoice are called sales discounts by the buyer.

13. If merchandise costing $2,500, terms 2/10 n/30, is paid within 10 days, the amount of the purchase
discount is $250.
Solution: $2,500 x.02 = $50.00

14. A quantity discount is recorded separately, the same way as a purchase discount.

Copyright © 2017 John Wiley & Sons Canada, Ltd. Unauthorized copying, distribution, or transmission of this page is prohibited
Test
5 -Bank
10 for Financial Accounting: Tools for Business Decision-Making, Seventh Canadian Edition

15. If a quantity discount of 10% is received on a purchase of $10,000, inventory would be recorded at $9,000.
Solution: $10,000 x.90 = $9,000

16. The Sales Returns and Allowances account and the Sales Discounts account are both classified as
expense accounts.

17. When goods are shipped FOB shipping point, freight costs are an operating expense for the seller.

18. When the terms of sale include a sales discount, it usually is advisable for the buyer to pay within the
discount period.

19. Merchandise is sold for $2,500 with terms 1/10, n/30. If $500 of the merchandise is returned prior to
payment and the invoice is paid within the discount period, the amount of the sales discount is $20.
Solution: ($2,500 – $500) x.01 = $20.00

20. When returned merchandise is defective, the seller’s sales account is debited.

21. The multiple-step income statement is considered more useful than the single-step income statement for a
merchandising company because it highlights the components of net income.

22. Operating expenses are similar in merchandising and service companies.

23. Gross profit appears on both the single-step and multiple-step forms of the income statement.

24. Non-operating activities include revenues and expenses that are related to the company’s main operations.

25. Corporations following IFRS must classify their expenses either by nature or by function.

26. Income from operations appears on both the single-step and multiple-step forms of the income statement.

27. A merchandising company’s income from operations is determined by subtracting cost of goods sold from
net sales.

28. Interest revenue for a merchandising company is usually reported in the non-operating activities section of
the income statement.

29. Companies following ASPE may classify their expenses by nature, but not by function.
Copyright © 2017 John Wiley & Sons Canada, Ltd. Unauthorized copying, distribution, or transmission of this page is prohibited
5 - 11 Merchandising Operations

30. Gross profit margin is the same as the gross profit amount.

31. If net sales are $1,000,000 and cost of goods sold is $800,000, the gross profit margin is 20%.
Solution: ($1,000,000 – $800,000) / $1,000,000 = 20%

32. The gross profit amount is generally considered to be more informative than the gross profit margin.

33. Gross profit margin is calculated by dividing cost of goods sold by net sales.

34. Profit margin indicates whether a company is controlling operating expenses relative to sales.

35. Profit margin is calculated by dividing net income by net sales.

*36. Purchase Returns and Allowances and Purchase Discounts are contra expense accounts with normal
credit balances.

*37. Freight In is subtracted from the Purchases account to arrive at cost of goods purchased.

*38. A key difference between the periodic and perpetual inventory systems is the timing of the calculation of
cost of goods sold.

*39. The cost of goods sold section of an income statement prepared under a periodic inventory system will
contain more detail than under a perpetual inventory system.

*40. On the income statement for a company using the periodic inventory system, the inventory at the
beginning of the period is added to the cost of merchandise purchased for the period to calculate the cost of
goods available for sale during the period.

*41. Compared to a perpetual inventory system, the use of the periodic inventory system will result in a
different value for inventory on the statement of financial position.

Copyright © 2017 John Wiley & Sons Canada, Ltd. Unauthorized copying, distribution, or transmission of this page is prohibited
Test
5 -Bank
12 for Financial Accounting: Tools for Business Decision-Making, Seventh Canadian Edition

ANSWERS TO TRUE-FALSE STATEMENTS

Item Ans. Item Ans. Item Ans. Item Ans. Item Ans.
1. T 10. F 19. T 28. T *37. F
2. F 11. F 20. F 29. F *38. T
3. T 12. F 21. T 30. F *39. T
4. T 13. F 22. T 31. T *40. T
5. F 14. F 23. F 32. F *41. F
6. T 15. T 24. F 33. F
7. F 16. F 25. T 34. T
8. F 17. F 26. F 35. T
9. T 18. T 27. F *36. T

Copyright © 2017 John Wiley & Sons Canada, Ltd. Unauthorized copying, distribution, or transmission of this page is prohibited
5 - 13 Merchandising Operations

MULTIPLE CHOICE QUESTIONS

42. Gross profit equals


(a) sales less operating expenses.
(b) gross sales less cost of goods sold.
(c) cost of goods sold less operating expenses.
(d) net sales less cost of goods sold.

43. The time it takes to go from cash to cash in producing revenues is called the
(a) accounting cycle.
(b) purchasing cycle.
(c) operating cycle.
(d) merchandising cycle.

44. Gross profit equals the difference between net sales and
(a) net income.
(b) cost of goods sold.
(c) operating expenses.
(d) cost of goods sold plus operating expenses.

45. Each of the following companies is a merchandising company except a


(a) wholesale parts company.
(b) candy store.
(c) moving company.
(d) furniture store.

46. Net income will result if gross profit exceeds


(a) cost of goods sold.
(b) operating expenses.
(c) purchases.
(d) cost of goods sold plus operating expenses.

47. A merchandiser will have income from operations of exactly $0 when


(a) net sales equals cost of goods sold.
(b) cost of goods sold equals gross profit.
(c) operating expenses equal net sales.
(d) gross profit equals operating expenses.

48. The largest current asset for a merchandiser is usually


(a) inventory.
(b) prepaid expenses.
(c) cash.
(d) accounts receivable.

Copyright © 2017 John Wiley & Sons Canada, Ltd. Unauthorized copying, distribution, or transmission of this page is prohibited
Test
5 -Bank
14 for Financial Accounting: Tools for Business Decision-Making, Seventh Canadian Edition

49. The primary source of revenue for a wholesaler is generated by


(a) investments.
(b) providing services.
(c) the sale of merchandise.
(d) the sale of property, plant, and equipment the company owns.

50. Generally, the revenue account for a merchandising company is called


(a) Sales Revenue or Sales.
(b) Investment Revenue.
(c) Gross Profit.
(d) Net Sales.

51. The operating cycle of a merchandising company is


(a) always one year in length.
(b) generally longer than that of a service company.
(c) about the same as that of a service company.
(d) generally shorter than that of a service company.

52. Net sales less cost of goods sold is called


(a) gross profit.
(b) cost of goods sold.
(c) net income.
(d) income before income taxes.

53. After gross profit is calculated, operating expenses are deducted to determine
(a) gross margin.
(b) net income (loss) before income tax.
(c) cost of goods sold.
(d) profit margin.

54. Which of the following “formulas” is incorrect?


(a) Gross profit – operating expenses = income before income tax.
(b) Net sales – cost of goods sold = gross profit.
(c) Net sales – gross profit = cost of goods sold.
(d) Operating expenses – cost of goods sold = gross profit.

55. Beginning inventory plus purchases equals


(a) cost of goods available for sale.
(b) cost of goods sold.
(c) ending inventory.
(d) total inventory on hand.

56. Which of the following is true about inventory systems?


(a) Periodic inventory systems require more detailed inventory records.
Copyright © 2017 John Wiley & Sons Canada, Ltd. Unauthorized copying, distribution, or transmission of this page is prohibited
5 - 15 Merchandising Operations

(b) Perpetual inventory systems require more detailed inventory records.


(c) A periodic system requires cost of goods sold to be recorded after each sale.
(d) A perpetual system determines cost of goods sold only at the end of the accounting period.

57. In a perpetual inventory system, cost of goods sold is recorded


(a) on a daily basis.
(b) on a monthly basis.
(c) on an annual basis.
(d) each time a sale occurs.

58. The primary difference between a periodic and a perpetual inventory system is that a periodic system
(a) keeps a detailed record showing the inventory on hand at all times.
(b) provides better control over inventories.
(c) records the cost of goods sold on the date the sale is made.
(d) determines the cost of goods sold at the end of the accounting period.

59. The physical inventory count is used to determine


(a) cost of inventory purchased during the period.
(b) cost of inventory sold during the period.
(c) the cost of inventory on hand.
(d) the cost of goods available for sale.

60. Inventory becomes part of the cost of goods sold when a company
(a) pays for the inventory.
(b) purchases the inventory.
(c) sells the inventory.
(d) receives payment from the customer.

61. If a company determines cost of goods sold each time a sale occurs, it
(a) must have a computerized accounting system.
(b) uses a combination of the perpetual and periodic inventory systems.
(c) uses a periodic inventory system.
(d) uses a perpetual inventory system.

62. Under a perpetual inventory system


(a) there is no need for a year-end physical count.
(b) increases in inventory resulting from purchases are debited to Purchases.
(c) accounting records continuously disclose the amount of inventory.
(d) the account Purchase Returns and Allowances is credited when goods are returned to vendors.

63. Under a perpetual inventory system, the following is determined each time a sale occurs:
(a) Gross Profit.
(b) Cost of Goods Sold.
(c) Purchases.
(d) Accounts Receivable.
Copyright © 2017 John Wiley & Sons Canada, Ltd. Unauthorized copying, distribution, or transmission of this page is prohibited
Test
5 -Bank
16 for Financial Accounting: Tools for Business Decision-Making, Seventh Canadian Edition

64. Under the perpetual inventory system, which of the following accounts would not be used?
(a) Sales
(b) Purchases
(c) Cost of Goods Sold
(d) Inventory

65. Given a perpetual inventory system which one of the following statements is false?
(a) Freight costs incurred by the buyer are added to the Inventory account.
(b) Purchases of merchandise for sale are recorded in the Inventory account.
(c) A discount taken for early payment is credited to the Inventory account.
(d) A return of merchandise is credited to the Purchase Returns and Allowances account.

66. The abbreviation "FOB" stands for


(a) free on board.
(b) freight on board.
(c) free only (to) buyer.
(d) freight charge on buyer.

67. On July 10, Arbour Inc. purchased $5,000 of inventory on terms of 2/10, n/30. The amount due on August
25 is
(a) $5,100.
(b) $5,000.
(c) $4,900.
(d) $4,990.

68. Under a perpetual inventory system, purchase of inventory is recorded as a debit to the
(a) Supplies account.
(b) Purchases account.
(c) Inventory account.
(d) Cost of Goods Sold account.

69. The journal entry by the buyer to record a return of merchandise purchased on account under a perpetual
inventory system would credit
(a) Accounts Payable.
(b) Purchase Returns and Allowances.
(c) Sales.
(d) Inventory.

70. A company using a perpetual inventory system that returns goods purchased on credit would
(a) debit Accounts Payable and credit Inventory.
(b) debit Sales and credit Accounts Payable.
(c) debit Cash and credit Accounts Payable.
(d) debit Accounts Payable and credit Purchases.

Copyright © 2017 John Wiley & Sons Canada, Ltd. Unauthorized copying, distribution, or transmission of this page is prohibited
5 - 17 Merchandising Operations

71. If a purchaser using a perpetual inventory system pays freight costs, then the
(a) Inventory account is increased.
(b) Inventory account is not affected.
(c) Freight Out account is increased.
(d) Freight In account is increased.

72. Freight costs incurred by a seller on merchandise sold to customers will cause an increase
(a) in the selling expenses of the buyer.
(b) in operating expenses for the seller.
(c) to the cost of goods sold of the seller.
(d) to a contra revenue account of the seller.

73. Alliance Corporation purchased inventory with an invoice price of $22,000 and credit terms of 1/10, n/15.
How much cash will Alliance pay if they pay within the discount period?
(a) $22,000
(b) $21,780
(c) $22,220
(d) $18,700
Solution: $22,000 x.99 = $21,780

74. For a company using a perpetual inventory system, the journal entry to record the purchase of $3,500 of
goods on account, with terms of 4/10, n/30, would include a
(a) debit to Accounts Payable of $3,500.
(b) credit to Accounts Payable of $3,360.
(c) debit to Inventory of $3,360.
(d) debit to Inventory of $3,500.

75. A purchase invoice is a document that


(a) provides support for goods sold for cash.
(b) provides evidence of operating expenses incurred.
(c) provides evidence of credit purchases.
(d) serves only as a customer receipt.

76. Sales Allowances and Sales Discounts


(a) are both designed to encourage customers to pay their accounts promptly.
(b) are both contra revenue accounts to Sales.
(c) both have a normal credit balance.
(d) both have a normal debit balance and are therefore regarded as expense accounts.

77. Under the perpetual inventory system, in addition to making the entry to record the sale, the seller would
(a) debit Inventory and credit Cost of Goods Sold.
(b) debit Cost of Goods Sold and credit Purchases.
(c) debit Cost of Goods Sold and credit Inventory.
(d) make no additional entry until the end of the period.

Copyright © 2017 John Wiley & Sons Canada, Ltd. Unauthorized copying, distribution, or transmission of this page is prohibited
Test
5 -Bank
18 for Financial Accounting: Tools for Business Decision-Making, Seventh Canadian Edition

78. Sales revenues are usually considered earned when


(a) cash is received from credit sales.
(b) an order is received.
(c) goods have been transferred from the seller to the buyer.
(d) adjusting entries are made.

79. Sales Discounts is a(n)


(a) contra revenue account.
(b) contra asset account.
(c) revenue account.
(d) expense account.

80. Evidence of cash sales is usually supported by


(a) purchase invoices.
(b) sales invoices.
(c) purchase orders.
(d) cash register tapes.

81. Gross sales less sales returns and allowances less sales discounts equals
(a) collectible sales.
(b) net sales.
(c) total sales.
(d) operating sales.

82. The entry to record a sale of $525 with terms of 2/10, n/30 will include a
(a) debit to Sales Discounts for $10.50.
(b) debit to Sales for $514.50.
(c) credit to Accounts Receivable for $525.
(d) credit to Sales for $525.

83. A sales invoice is prepared when goods


(a) are sold for cash.
(b) are sold on credit.
(c) sold on credit are returned.
(d) are sold on credit or for cash.

84. Sales Returns and Allowances is a(n)


(a) asset account.
(b) contra asset account.
(c) expense account.
(d) contra revenue account.

85. The entry to record the return of goods from a customer would include a
(a) debit to Sales.
Copyright © 2017 John Wiley & Sons Canada, Ltd. Unauthorized copying, distribution, or transmission of this page is prohibited
5 - 19 Merchandising Operations

(b) credit to Sales.


(c) debit to Sales Returns and Allowances.
(d) credit to Sales Returns and Allowances.

86. The collection of a $2,000 account within the 2 percent discount period will result in a
(a) debit to Sales Discounts for $40.
(b) debit to Accounts Receivable for $1,960.
(c) credit to Cash for $1,960.
(d) credit to Accounts Receivable for $1,960.
Solution: $2,000 x.02 = $40

87. Freight paid by the seller to a customer’s business is recorded as a


(a) credit to Sales.
(b) debit to Sales.
(c) debit to an operating expense.
(d) credit to Cost of Goods Sold.

88. If a customer agrees to keep defective merchandise because the seller is willing to reduce the selling price,
this transaction is known as a sales
(a) discount.
(b) return.
(c) contra asset.
(d) allowance.

89. When goods from a cash sale are returned, the effect on the seller’s accounts will be
(a) an increase in net sales.
(b) a decrease in gross sales.
(c) an increase in gross sales.
(d) a decrease in net sales.

90. Management may be alerted to a quality problem with their merchandise by a sudden increase in which
account?
(a) Sales
(b) Sales Returns and Allowances
(c) Sales Discounts
(d) Cost of Goods Sold

91. A Sales Returns and Allowances account is not debited if a customer


(a) returns defective merchandise.
(b) receives a credit for merchandise of inferior quality.
(c) pays within the discount period.
(d) returns goods that are not in accordance with specifications.

92. As an incentive for customers to pay their accounts promptly, a business may offer its customers
(a) a sales discount.
Copyright © 2017 John Wiley & Sons Canada, Ltd. Unauthorized copying, distribution, or transmission of this page is prohibited
Test
5 -Bank
20 for Financial Accounting: Tools for Business Decision-Making, Seventh Canadian Edition

(b) free delivery.


(c) a sales allowance.
(d) a sales return.

93. The credit terms offered by a company are 2/10, n/30, which means that
(a) the customer must pay the bill within 10 days.
(b) the customer can deduct a 2% discount if the bill is paid between 10 days and 30 days from the invoice
date.
(c) the customer can deduct a 2% discount if the bill is paid within 10 days of the invoice date.
(d) two sales returns can be made within 10 days of the invoice date and no returns thereafter.

94. A sales discount does not


(a) provide the purchaser with a cash saving.
(b) reduce the amount of cash received from a credit sale.
(c) increase a contra revenue account.
(d) increase an operating expense account.

95. Alpha Corp. sells $1,000 of merchandise on account to Beta Corp. with credit terms of 2/10, n/30. If Beta
pays within the discount period, how much cash will Alpha receive?
(a) $900
(b) $920
(c) $980
(d) $1,000
Solution: $1,000 x.98 = $980

96. Mindful Corporation sells merchandise on account for $5,000 to Absent Corporation with credit terms of
2/10, n/30. Absent returns $750 of merchandise that was damaged, along with a cheque to settle the account
within the discount period. What is the amount of the cheque?
(a) $4,900
(b) $4,250
(c) $5,000
(d) $4,165
Solution: ($5,000 – $750) x.98 = $4,165

97. Raven Corp. sells merchandise on account for $4,000 to Eagle Corp., terms 2/10, n/30. Eagle returns
$1,600 worth of merchandise that was damaged, along with a cheque to settle the account within the discount
period. What entry does Raven make upon receipt of the cheque?
(a) Cash............................................................................................ 2,400
Accounts Receivable............................................................ 2,400
(b) Cash............................................................................................ 2,320
Sales Returns and Allowances.................................................... 1,568
Sales Discounts........................................................................... 64
Accounts Receivable............................................................ 4,000
(c) Cash............................................................................................ 2,352
Sales Returns and Allowances.................................................... 1,600
Sales Discounts........................................................................... 48
Accounts Receivable............................................................ 4,000
Copyright © 2017 John Wiley & Sons Canada, Ltd. Unauthorized copying, distribution, or transmission of this page is prohibited
5 - 21 Merchandising Operations

(d) Cash............................................................................................ 2,320


Sales Discounts........................................................................... 80
Sales Returns and Allowances.................................................... 1,600
Accounts Receivable............................................................ 4,000
Solution: Cash: ($4,000 – $1,600) x.98 = $2,352; Discount: ($4,000 – $1,600) x.02 = $48

98. The collection of a $1,000 account paid within the 2 percent discount period will result in a
(a) credit to Cash for $980.
(b) credit to Accounts Receivable for $1,000.
(c) debit to Cash for $1,000.
(d) credit to Accounts Receivable for $980.

99. Which of the following would not be classified as a contra account?


(a) Sales
(b) Sales Returns and Allowances
(c) Accumulated Depreciation
(d) Sales Discounts

100. Which of the following accounts has a normal credit balance?


(a) Sales Returns and Allowances
(b) Sales Discounts
(c) Sales
(d) Cost of Goods Sold

101. The respective normal account balances of Sales, Sales Returns and Allowances, and Sales Discounts
are
(a) credit, credit, credit.
(b) debit, credit, debit.
(c) credit, debit, debit.
(d) credit, debit, credit.

102. Which one of the following would not appear on a single-step income statement?
(a) gross profit
(b) expenses
(c) sales revenues
(d) cost of goods sold

103. The form of the income statement that derives its name from the fact that the total of all expenses is
deducted from the total of all revenues is called a
(a) multiple-step income statement.
(b) revenue income statement.
(c) report-form income statement.
(d) single-step income statement.

104. Gross profit does not appear


Copyright © 2017 John Wiley & Sons Canada, Ltd. Unauthorized copying, distribution, or transmission of this page is prohibited
Test
5 -Bank
22 for Financial Accounting: Tools for Business Decision-Making, Seventh Canadian Edition

(a) on a multiple-step income statement.


(b) on a single-step income statement.
(c) to be relevant in analyzing the operation of a merchandising company.
(d) on either a multiple-step or a single-step income statement.

105. Gross profit for a merchandising company equals the difference between net sales and
(a) operating expenses.
(b) cost of goods sold.
(c) net income.
(d) cost of goods sold plus operating expenses.

106. A loss from operations will result if operating expenses exceed


(a) cost of goods sold.
(b) selling expenses.
(c) cost of goods sold plus sales returns and allowances.
(d) gross profit.

107. What is the term applied to the excess of net sales over the cost of goods sold?
(a) gross sales
(b) income from operations
(c) net income
(d) gross profit

108. Which of the following is not true about a multiple-step income statement?
(a) There is a section for operating expenses.
(b) There may be a section for non-operating activities.
(c) There may be a section for operating assets.
(d) There is a section for cost of goods sold.

109. An advantage of the single-step income statement over the multiple-step form is
(a) the amount of information it provides.
(b) its comprehensiveness.
(c) its simplicity.
(d) its use in calculating ratios.

110. Income from operations appears on


(a) both a multiple-step and a single-step income statement.
(b) neither a multiple-step nor a single-step income statement.
(c) a single-step income statement only.
(d) a multiple-step income statement only.

111. Which statement is correct about expenses on the income statement?


(a) Classifying expenses by nature means that expenses are reported according to the activity for which they
are incurred.
(b) Examples of expenses classified by function are cost of goods sold and administrative expenses.
Copyright © 2017 John Wiley & Sons Canada, Ltd. Unauthorized copying, distribution, or transmission of this page is prohibited
5 - 23 Merchandising Operations

(c) Expenses must be classified by their function.


(d) Expenses must be classified in decreasing order of magnitude.

112. Which statement is not correct about expenses on the income statement?
(a) Classifying expenses by function means that expenses are reported according to the activity for which they
are incurred.
(b) Examples of expenses classified by nature are salaries and depreciation.
(c) Companies following ASPE do not have to list their expenses in any particular order.
(d) Expenses must be classified in decreasing order of magnitude.

113. A multiple-step income statement shows


(a) gross profit but not income from operations.
(b) neither gross profit nor income from operations.
(c) both gross profit and income from operations.
(d) income from operations but not gross profit.

114. Interest expense would be classified on a multiple-step income statement under the heading
(a) Other expenses and losses.
(b) Other revenues and gains.
(c) Operating expenses.
(d) Cost of goods sold.

115. Income from operations for a merchandising company is net sales less
(a) operating expenses.
(b) cost of goods sold.
(c) sales discounts and cost of goods sold.
(d) operating expenses and cost of goods sold.

116. The operating expenses section of a multiple-step income statement for a merchandising company would
not include
(a) freight out.
(b) utilities expense.
(c) cost of goods sold.
(d) loss on sale of equipment.

117. Which one of the following would appear on the income statement of both a merchandising company and
a service company?
(a) Gross profit
(b) Net income
(c) Sales revenues
(d) Cost of goods sold

118. Gross profit does not appear


(a) on a merchandising company’s multiple-step income statement.
(b) on a service company’s income statement.
Copyright © 2017 John Wiley & Sons Canada, Ltd. Unauthorized copying, distribution, or transmission of this page is prohibited
Test
5 -Bank
24 for Financial Accounting: Tools for Business Decision-Making, Seventh Canadian Edition

(c) to be relevant in analyzing the operation of a merchandising company.


(d) on the income statement, if the periodic inventory system is used, because it cannot be calculated.

Use the following information to answer questions 119–122.


Cost of goods sold.............................................................. $434,000
Income tax expense........................................................... 67,200
Operating expenses........................................................... 344,000
Sales.................................................................................. 1,100,000
Sales discounts.................................................................. 24,000
Sales returns and allowances............................................. 74,000

119. The amount of net sales on the income statement would be


(a) $1,002,000.
(b) $1,076,000.
(c) $1,026,000.
(d) $1,100,000.
Solution: ($1,100,000 – $24,000 – $74,000) = $1,002,000

120. Gross profit would be


(a) $224,000.
(b) $568,000.
(c) $756,000.
(d) $1,002,000.
Solution: ($1,002,000 – $434,000) = $568,000

121. The gross profit margin would be


(a) 56.7%.
(b) 34.3%.
(c) 43.3%.
(d) 39.5%.
Solution: $568,000 / $1,002,000 = 56.7%

122. The profit margin would be


(a) 18.5%.
(b) 15.6%.
(c) 60.6%.
(d) 34.3%.
Solution: ($568,000 – $344,000 – $67,200) / $1,002,000 = 15.6%

123. Gross profit is


(a) a measure of the overall net income of a company.
(b) is expressed as a percentage of gross sales.
(c) also called gross margin.
(d) the same as gross profit margin.

Copyright © 2017 John Wiley & Sons Canada, Ltd. Unauthorized copying, distribution, or transmission of this page is prohibited
5 - 25 Merchandising Operations

124. The gross profit margin is calculated by dividing gross profit by


(a) sales.
(b) cost of goods sold.
(c) net sales.
(d) operating expenses.

125. A decline in a company’s gross profit could be caused by all of the following except
(a) selling products with a lower markup.
(b) clearance of discontinued inventory.
(c) paying lower prices to its suppliers.
(d) increased competition resulting in lower selling prices.

126. If a company has net sales of $500,000 and cost of goods sold of $350,000, the gross profit margin is
(a) 15%.
(b) 30%.
(c) 70%.
(d) 100%.
Solution: ($500,000 – $350,000) / $500,000 = 30%

127. A company shows the following balances:


Cost of goods sold.............................................................. $ 900,000
Sales.................................................................................. 2,000,000
Sales discounts.................................................................. 25,000
Sales returns and allowances............................................. 225,000
What is the gross profit margin?
(a) 42.5%
(b) 48.6%
(c) 49.3%
(d) 55.0%
Solution: ($2,000,000 – $25,000 – $225,000 – $900,000) / ($2,000,000 – $25,000 – $225,000) = 48.6%

128. Profit margin is calculated by dividing


(a) net income by gross profit.
(b) net income by sales.
(c) net income by net sales.
(d) sales by net income.

129. Profit margin is a measure of


(a) liquidity.
(b) profitability.
(c) solvency.
(d) comparability.

130. Profit margin is calculated by dividing net income by


(a) sales.
(b) sales revenues.
Copyright © 2017 John Wiley & Sons Canada, Ltd. Unauthorized copying, distribution, or transmission of this page is prohibited
Test
5 -Bank
26 for Financial Accounting: Tools for Business Decision-Making, Seventh Canadian Edition

(c) net sales.


(d) gross sales.

Use the following financial information to answer questions 131–132.

Operating expenses........................................................... $ 25,000


Sales returns and allowances............................................. 3,000
Sales.................................................................................. 110,000
Cost of goods sold.............................................................. 55,000
Income tax expense........................................................... 5,000

131. What is the gross profit margin?


(a) 20.6%
(b) 22.7%
(c) 48.6%
(d) 50.0%
Solution: ($110,000 – $3,000 – $55,000) / ($110,000 – $3,000) = 48.6%

132. What is the profit margin?


(a) 20.6%
(b) 22.7%
(c) 48.6%
(d) 50.0%
Solution: ($110,000 – $3,000 – $55,000 – $25,000 – $5,000) / ($110,000 – $3,000) = 20.6%

*133. Under a periodic inventory system,


(a) purchases of inventory are debited to the Purchases account.
(b) freight incurred on merchandise purchases by the buyer should be debited to the Inventory account.
(c) purchases of merchandise are usually credited to the Purchases account.
(d) freight incurred on merchandise sales by the seller should be debited to the Freight In account.

*134. Which of the following is not true for a company using a periodic inventory system?
(a) Cost of goods sold is calculated for each sale.
(b) Cost of goods sold is calculated at the end of the accounting period.
(c) A physical inventory count is performed at the end of the accounting period.
(d) Cost of goods available for sale is calculated at the end of the accounting period.

*135. Detailed records of goods held for resale are not maintained under a
(a) perpetual inventory system.
(b) periodic inventory system.
(c) double entry accounting system.
(d) single entry accounting system.

*136. Purchases less purchase returns and allowances less purchase discounts is called
(a) cost of goods purchased.
Copyright © 2017 John Wiley & Sons Canada, Ltd. Unauthorized copying, distribution, or transmission of this page is prohibited
5 - 27 Merchandising Operations

(b) net purchases.


(c) cost of goods sold.
(d) net inventory.

*137. Under a periodic inventory system, purchase of merchandise is debited to the


(a) Inventory account.
(b) Cost of Goods Sold account.
(c) Purchases account.
(d) Accounts Payable account.

*138. Which of the following accounts has a normal credit balance?


(a) Purchases
(b) Sales Returns and Allowances
(c) Freight In
(d) Purchase Discounts

*139. The respective normal balances of Purchases, Purchase Discounts, and Freight In are
(a) credit, credit, debit.
(b) debit, credit, credit.
(c) debit, credit, debit.
(d) debit, debit, debit.

*140. The Freight In account


(a) increases the cost of merchandise purchased.
(b) is a contra account to the Purchases account.
(c) is a permanent account.
(d) has a normal credit balance.

*141. Net purchases plus freight in is called


(a) cost of goods sold.
(b) cost of goods available for sale.
(c) cost of goods purchased.
(d) total goods available for sale.

*142. Beginning inventory plus the cost of goods purchased equals


(a) cost of goods sold.
(b) cost of goods available for sale.
(c) net purchases.
(d) total goods purchased.

*143. On the income statement, purchases less purchase discounts and purchase returns and allowances,
plus freight in equals
(a) cost of goods purchased.
(b) cost of goods available for sale.
(c) net purchases.
Copyright © 2017 John Wiley & Sons Canada, Ltd. Unauthorized copying, distribution, or transmission of this page is prohibited
Test
5 -Bank
28 for Financial Accounting: Tools for Business Decision-Making, Seventh Canadian Edition

(d) gross profit.

*144. Market Inc. shows the following account balances for last month:
Freight In............................................................................ $ 3,750
Freight Out......................................................................... 2,425
Purchases.......................................................................... 33,500
Purchase Discounts........................................................... 1,005
Sales Returns and Allowances........................................... 4,250
The cost of goods purchased for last month is
(a) $33,820.
(b) $36,245.
(c) $32,495.
(d) $33,500.
Solution: ($33,500 + $3,750 – $1,005) = $36,245

*145. ClearEyes Inc. reported beginning inventory of $20,000. During the period, purchases were $140,000;
purchase returns, $4,000; and freight in $10,000. A physical count of inventory at the end of the period
revealed that $30,000 was still on hand. The cost of goods available for sale was
(a) $156,000.
(b) $164,000.
(c) $166,000.
(d) $184,000.
Solution: $20,000 + (140,000 – 4,000 + 10,000) = $166,000

*146. Cost of goods sold is calculated from the following equation:


(a) beginning inventory – cost of goods purchased + ending inventory.
(b) sales – cost of goods purchased + beginning inventory – ending inventory.
(c) sales + gross profit – ending inventory + beginning inventory.
(d) beginning inventory + cost of goods purchased – ending inventory.

Use the following information to answer questions *147–*149.

For last month, the following data were taken from the ledger of Rockit Inc.:
Beginning Inventory............................................................ $ 43,000
Ending Inventory................................................................ 32,400
Freight In............................................................................ 2,300
Purchases.......................................................................... 224,000
Purchase Discounts........................................................... 1,500
Purchase Returns and Allowances..................................... 3,800

*147. What was the cost of goods purchased?


(a) $220,200
(b) $218,700
(c) $221,000
(d) $216,400
Solution: $224,000 – 1,500 – 3,800 + 2,300 = $221,000

Copyright © 2017 John Wiley & Sons Canada, Ltd. Unauthorized copying, distribution, or transmission of this page is prohibited
5 - 29 Merchandising Operations

*148. What was the cost of goods sold?


(a) $234,600
(b) $231,600
(c) $237,600
(d) $213,400
Solution: $43,000 + ($224,000 – 1,500 – 3,800 + 2,300) – $32,000 = $231,600

*149. What was the cost of goods available for sale?


(a) $264,000
(b) $267,000
(c) $269,300
(d) $234,600
Solution: $43,000 + ($224,000 – 1,500 – 3,800 + 2,300) = $264,000

*150. On the income statement, the beginning inventory is added to the cost of goods purchased to determine
the
(a) cost of goods sold.
(b) cost of goods available for sale.
(c) income from operations.
(d) gross profit.

Copyright © 2017 John Wiley & Sons Canada, Ltd. Unauthorized copying, distribution, or transmission of this page is prohibited
Test
5 -Bank
30 for Financial Accounting: Tools for Business Decision-Making, Seventh Canadian Edition

ANSWERS TO MULTIPLE CHOICE QUESTIONS

Item Ans. Item Ans. Item Ans. Item Ans. Item Ans. Item Ans. Item Ans.
42. d 58. d 74. d 90. b 106. d 122. b *138. d
43. c 59. c 75. c 91. c 107. d 123. c *139. c
44. b 60. c 76. b 92. a 108. c 124. c *140. a
45. c 61. d 77. c 93. c 109. c 125. c *141. c
46. b 62. c 78. c 94. d 110. d 126. b *142. b
47. d 63. b 79. a 95. c 111. b 127. b *143. a
48. a 64. b 80. d 96. d 112. d 128. c *144. b
49. c 65. d 81. b 97. c 113. c 129. b *145. c
50. a 66. a 82. d 98. b 114. a 130. c *146. d
51. b 67. b 83. b 99. a 115. d 131. c *147. c
52. a 68. c 84. d 100. c 116. c 132. a *148. b
53. b 69. d 85. c 101. c 117. b *133. a *149. a
54. d 70. a 86. a 102. a 118. b *134. a *150. b
55. a 71. a 87. c 103. d 119. a *135. b
56. b 72. b 88. d 104. b 120. b *136. b
57. d 73. b 89. d 105. b 121. a *137. c

Copyright © 2017 John Wiley & Sons Canada, Ltd. Unauthorized copying, distribution, or transmission of this page is prohibited
5 - 31 Merchandising Operations

EXERCISES

Ex. 151
Ann-Marie Carver is a new accountant with Ornell Corporation. Ornell purchased merchandise on account for
$10,000. The credit terms are 2/10, n/30. Ann-Marie has talked with the company's banker and knows that she
could earn 10% on any money invested in the company's savings account.

Instructions
(a) Should Ann-Marie pay the invoice within the discount period or should she keep the $10,000 in the
savings account and pay at the end of the credit period (i.e., after 30 days)? Support your
recommendation with a calculation showing which action would be best.
(b) If Ann-Marie forgoes the discount, it may be viewed as paying an interest rate of 2% for the use of $10,000
for 20 days. Calculate the annual rate of interest that this is equivalent to.

Solution 151 (10 min.)


(a) Discount of 2% on $10,000 $200.00
Interest received on $10,000 (for 20 days at 10%)$54.79 ($10,000  10%  20  365)
Savings by taking the discount $145.21
Recommendation: Ann-Marie should pay the invoice within the discount period.

(b) The equivalent annual interest rate is:


2%  365  20 = 36.5%.

Ex. 152
Magnesium Inc. uses a perpetual inventory system. During April, the following transactions occurred:
Apr 3 Purchased $2,000 of merchandise, terms 3/10, n/60.
6 Returned $300 of the merchandise purchased on April 3.
7 Paid freight charges of $150 on goods purchased on April 3.
12 Paid for the goods purchased on April 3.
13 Sold goods costing $600 on credit for $1,000, terms 2/10, n/45.
14 The customer of April 13 returned $300 of the goods that had a cost of $180.
23 Received payment from the customer of April 13.

Instructions
Prepare journal entries to record the above transactions.

Solution 152
Apr 3 Inventory............................................................................ 2,000
Accounts Payable........................................................ 2,000

6 Accounts Payable............................................................... 300


Inventory..................................................................... 300

7 Inventory............................................................................ 150
Cash............................................................................ 150

12 Accounts Payable ($2,000 – $300)..................................... 1,700


Copyright © 2017 John Wiley & Sons Canada, Ltd. Unauthorized copying, distribution, or transmission of this page is prohibited
Test
5 -Bank
32 for Financial Accounting: Tools for Business Decision-Making, Seventh Canadian Edition

Inventory ($1,700 x 3%).............................................. 51


Cash ($1,700 x 97%)................................................... 1,649

13 Accounts Receivable.......................................................... 1,000


Sales........................................................................... 1,000

Cost of Goods Sold............................................................ 600


Inventory..................................................................... 600

14 Sales Returns and Allowances........................................... 300


Accounts Receivable................................................... 300

Inventory............................................................................ 180
Cost of Goods Sold..................................................... 180

23 Cash ($700 x 98%)............................................................. 686


Sales Discounts ($700 x 2%)............................................. 14
Accounts Receivable ($1,000 – $300)......................... 700

Ex. 153
Jun 4 Willem Corporation purchased $4,000 worth of merchandise, terms 2/10, n/30 from Cate
Corporation. The cost of the merchandise to Cate was $2,600.
10 Willem returned $700 worth of goods to Cate for full credit. The goods had a cost of $450 to Cate
and were placed back into inventory.
26 Willem paid the account.

Instructions
Prepare the journal entries to record these transactions in (a) Willem’s records and (b) Cate’s records. Both
companies use the perpetual inventory system.

Solution 153 (15–20 min.)


(a) Willem’s records
Jun 4 Inventory............................................................................ 4,000
Accounts Payable........................................................ 4,000

10 Accounts Payable............................................................... 700


Inventory..................................................................... 700

26 Accounts Payable ($4,000 – $700)..................................... 3,300


Cash............................................................................ 3,300

(b) Cate’s records


Jun 4 Accounts Receivable.......................................................... 4,000
Sales........................................................................... 4,000

4 Cost of Goods Sold............................................................ 2,600


Inventory..................................................................... 2,600

10 Sales Returns and Allowance............................................. 700


Accounts Receivable................................................... 700
Copyright © 2017 John Wiley & Sons Canada, Ltd. Unauthorized copying, distribution, or transmission of this page is prohibited
5 - 33 Merchandising Operations

10 Inventory............................................................................ 450
Cost of Goods Sold..................................................... 450

26 Cash................................................................................... 3,300
Accounts Receivable ($4,000 – $700)......................... 3,300

Ex. 154
On July 1, Racquets Plus had an inventory of 20 tennis racquets at a cost of $125 each. Racquets Plus uses a
perpetual inventory system. During the month of July, the following transactions occurred:
Jul 4 Purchased 25 racquets at a cost of $125 each from the Tennis Gear Corporation, terms 2/10, n/30.
5 Paid freight of $100 on the July 4 purchase.
6 Sold 10 racquets from the July 1 inventory to Team Canada for $225 each, terms 2/10, n/30.
7 Received a credit from Tennis Gear for the return of 2 defective racquets.
8 Sold two racquets from the July 1 inventory for $550 cash.
13 Issued a credit memo to Team Canada for the return of a defective racquet.
14 Paid Tennis Gear in full.
15 Received payment from Team Canada.

Instructions
Record the July transactions for Racquets Plus.

Solution 154 (20 min.)


Jul 4 Inventory ($125 x 25)......................................................... 3,125
Accounts Payable........................................................ 3,125

5 Inventory............................................................................ 100
Cash............................................................................ 100

6 Accounts Receivable ($225 x 10)....................................... 2,250


Sales........................................................................... 2,250
Cost of Goods Sold ($125 x 10)......................................... 1,250
Inventory..................................................................... 1,250

7 Accounts Payable............................................................... 250


Inventory..................................................................... 250

8 Cash................................................................................... 550
Sales........................................................................... 550
Cost of Goods Sold ($125 x 2)........................................... 250
Inventory..................................................................... 250

13 Sales Returns and Allowances........................................... 225


Accounts Receivable................................................... 225

14 Accounts Payable ($3,125 – $250)..................................... 2,875


Cash ($2,875  98%)................................................... 2,817.50
Inventory ($2,875  2%).............................................. 57.50

Copyright © 2017 John Wiley & Sons Canada, Ltd. Unauthorized copying, distribution, or transmission of this page is prohibited
Test
5 -Bank
34 for Financial Accounting: Tools for Business Decision-Making, Seventh Canadian Edition

15 Cash ($2,025 x 98%).......................................................... 1,984.50


Sales Discounts ($2,025 x 2%).......................................... 40.50
Accounts Receivable ($2,250 – $225)......................... 2,025

Ex. 155
On September 1, Wilderness Inc. had an inventory of 18 backpacks at a cost of $30 each. The company uses
a perpetual inventory system. During September, the following transactions occurred:
Sep 4 Purchased 35 backpacks at $30 each from Back Packs Unlimited, terms 3/10, n/30.
6 Received credit of $150 for the return of 5 backpacks purchased on Sept. 4 that were defective.
9 Sold 20 backpacks for $50 each to University Supply, terms 2/10, n/30.
14 Paid Back Packs Unlimited in full.
18 Received payment from University Supply.

Instructions
Record the September transactions for Wilderness Inc.

Solution 155 (15–20 min.)


Sep 4 Inventory ($30 x 35)........................................................... 1,050
Accounts Payable........................................................ 1,050

6 Accounts Payable............................................................... 150


Inventory..................................................................... 150

9 Accounts Receivable ($50 x 20)......................................... 1,000


Sales........................................................................... 1,000

Cost of Goods Sold ($30 x 20)........................................... 600


Inventory..................................................................... 600

14 Accounts Payable ($1,050 – $150)..................................... 900


Cash ($900  97%)...................................................... 873
Inventory ($900  3%)................................................. 27

18 Cash ($1,000 x 98%).......................................................... 980


Sales Discounts ($1,000 x 2%).......................................... 20
Accounts Receivable................................................... 1,000

Ex. 156
Gia’s Gymnastics Gear uses a perpetual inventory system. The following transactions occurred in July:
Jul 6 Purchased $1,800 of merchandise on credit, terms 1/10, n/30.
8 Because some of the items purchased on July 6 had a small defect, Gia’s Gymnastics Gear
received a purchase allowance of $175.
9 Paid freight charges of $75 on the items purchased July 6.
19 Sold merchandise on credit for $1,800, terms 2/10, n/30. The merchandise had a cost of $900.
22 Of the merchandise sold on July 19, $200 of it was returned. The items had cost Gia’s$100 and
were returned to inventory.
28 Received payment from the customer of July 19.
Copyright © 2017 John Wiley & Sons Canada, Ltd. Unauthorized copying, distribution, or transmission of this page is prohibited
5 - 35 Merchandising Operations

31 Paid for the merchandise purchased on July 6.

Instructions
Record the July transactions for Gia’s Gymnastics Gear.

Solution 156 (15–20 min.)


Jul 6 Inventory............................................................................ 1,800
Accounts Payable........................................................ 1,800

8 Accounts Payable............................................................... 175


Inventory..................................................................... 175

9 Inventory............................................................................ 75
Cash............................................................................ 75

19 Accounts Receivable.......................................................... 1,800


Sales........................................................................... 1,800

Cost of Goods Sold............................................................ 900


Inventory..................................................................... 900

22 Sales Returns and Allowances........................................... 200


Accounts Receivable................................................... 200

Inventory............................................................................ 100
Cost of Goods Sold..................................................... 100

28 Cash ($1,600 x 98%).......................................................... 1,568


Sales Discounts ($1,600 x 2%).......................................... 32
Accounts Receivable ($1,800 – $200)......................... 1,600

31 Accounts Payable ($1,800 – $175)..................................... 1,625


Cash............................................................................ 1,625

Ex. 157
(a) Arbour Corporation purchased merchandise on account from Lavalle Supplies for $136,000, with terms of
2/10, n/30. During the discount period, Arbour returned some merchandise and paid $113,680 as payment
in full. Arbour uses a perpetual inventory system. Prepare the journal entries that Arbour made to record
the
1. purchase of merchandise.
2. return of merchandise.
3. payment on account.

(b) Ransak Corporation sold merchandise to Belville Corporation on account for $168,000 with credit terms
of?/10, n/30. The cost of the merchandise sold was $84,000. During the discount period, Belville returned
$28,000 worth of merchandise and paid its account in full (minus the return and the discount) by paying
$134,400 in cash. The returned goods were returned to inventory. Both companies use a perpetual
inventory system. Prepare the journal entries that Ransak Corporation made to record the
1. sale of merchandise.

Copyright © 2017 John Wiley & Sons Canada, Ltd. Unauthorized copying, distribution, or transmission of this page is prohibited
Test
5 -Bank
36 for Financial Accounting: Tools for Business Decision-Making, Seventh Canadian Edition

2. return of merchandise.
3. collection on account.

Solution 157 (15–20 min.)


(a) To calculate the amount due after returns but before the discount, divide $113,680 by 98% (100% – 2%) =
$113,680  98% = $116,000
Subtract $116,000 from $136,000 to determine that $20,000 of merchandise was returned.

1. Inventory................................................................................. 136,000
Accounts Payable............................................................ 136,000

2. Accounts Payable................................................................... 20,000


Inventory.......................................................................... 20,000

3. Accounts Payable................................................................... 116,000


Inventory (116,000 x 2%)................................................. 2,320
Cash................................................................................ 113,680

(b) Belville returns $28,000 of merchandise and thus owes $140,000 to Ransak.
$134,400  $140,000 = 96%; 100% – 96% = 4%
The missing discount percentage is 4%. $140,000  4% = $5,600 sales discount
$140,000 – $5,600 = $134,400 cash received on account

1. Accounts Receivable.............................................................. 168,000


Sales................................................................................ 168,000

Cost of Goods Sold................................................................. 84,000


Inventory.......................................................................... 84,000

2. Sales Returns and Allowances............................................... 28,000


Accounts Receivable........................................................ 28,000

Inventory................................................................................. 14,000
Cost of Goods Sold.......................................................... 14,000

3. Cash....................................................................................... 134,400
Sales Discounts...................................................................... 5,600
Accounts Receivable........................................................ 140,000

Ex. 158
On June 1, Grill Master Ltd. had an inventory of 10 barbeques at a cost of $440 each. Grill Master uses a
perpetual inventory system. During the month of June the following transactions occurred:
Jun 3 Purchased 25 barbeques at a cost of $440 each from BBQ King Ltd., terms n/30.
5 Paid $200 freight for the barbeques purchased on June 3.
6 Sold 12 barbeques to Outdoor Grill for $760 each, terms 2/10, n/30.
7 Received credit from BBQ King for the return of two defective barbeques.
13 Issued a credit to Outdoor Grill for the return of one defective barbeque.
16 Received a credit from BBQ King for the defective barbeque returned by Outdoor Grill.

Copyright © 2017 John Wiley & Sons Canada, Ltd. Unauthorized copying, distribution, or transmission of this page is prohibited
5 - 37 Merchandising Operations

19 Purchased 10 barbeques from Backyard Barbecues at a cost of $440 each, terms 2/10, n/30.
20 Paid freight of $200 on the June 19 purchase.

Instructions
Prepare journal entries to record the above transactions.

Solution 158
Jun 3 Inventory ($440 x 25)......................................................... 11,000
Accounts Payable........................................................ 11,000

5 Inventory............................................................................ 200
Cash............................................................................ 200

6 Accounts Receivable ($760 x 12)....................................... 9,120


Sales........................................................................... 9,120

Cost of Goods Sold ($440 x 12)......................................... 5,280


Inventory..................................................................... 5,280

7 Accounts Payable ($440 x 2).............................................. 880


Inventory..................................................................... 880

13 Sales Returns and Allowances........................................... 760


Accounts Receivable................................................... 760

Inventory............................................................................ 440
Cost of Goods Sold..................................................... 440

16 Accounts Payable............................................................... 440


Inventory..................................................................... 440

19 Inventory ($440 x 10)......................................................... 4,400


Accounts Payable........................................................ 4,400

20 Inventory............................................................................ 200
Cash............................................................................ 200

Ex. 159
Presented below are selected transactions for Manclave Corporation during July.
Jul 1 Sold merchandise to Regina Inc. for $1,000, terms 3/10, n/30. The merchandise sold cost $600.
2 Purchased merchandise from Novalle Corporation for $5,200, terms 4/10, n/30.
3 Paid freight charges of $150 on items purchased on July 2.
4 Purchased merchandise from Ollie Company Ltd. for $7,500, n/30.
10 Received payment from Regina Inc. for purchase of July 1.
11 Paid Novalle Corporation for July 2 purchase.

Instructions
(a) Record the above transactions for Manclave Corporation, assuming a perpetual inventory system is used.
The cost of goods sold on July 1 was determined to be $400.
Copyright © 2017 John Wiley & Sons Canada, Ltd. Unauthorized copying, distribution, or transmission of this page is prohibited
Test
5 -Bank
38 for Financial Accounting: Tools for Business Decision-Making, Seventh Canadian Edition

(b) Record the above transactions for Manclave Corporation, assuming a periodic inventory system is used.

Solution 159 (25 min.)


(a) Perpetual
Jul 1 Accounts Receivable.......................................................... 1,000
Sales......................................................................... 1,000

Cost of Goods Sold............................................................ 600


Inventory................................................................... 600

2 Inventory............................................................................ 5,200
Accounts Payable .................................................... 5,200

3 Inventory............................................................................ 150
Cash.......................................................................... 150

4 Inventory............................................................................ 7,500
Accounts Payable .................................................... 7,500

10 Cash ($1,000 x 97%).......................................................... 970


Sales Discounts ($1,000 x 3%).......................................... 30
Accounts Receivable................................................. 1,000

11 Accounts Payable............................................................... 5,200


Inventory ($5,200 x 4%)............................................ 208
Cash ($5,200 x 96%)................................................ 4,992

(b) Periodic
Jul 1 Accounts Receivable.......................................................... 1,000
Sales......................................................................... 1,000

2 Purchases.......................................................................... 5,200
Accounts Payable..................................................... 5,200

3 Freight in............................................................................ 150


Cash....................................................................... 150

4 Purchases.......................................................................... 7,500
Accounts Payable..................................................... 7,500

10 Cash ($1,000 x 97%).......................................................... 970


Sales Discounts ($1,000 x 3%).......................................... 30
Accounts Receivable................................................. 1,000

11 Accounts Payable............................................................... 5,200


Purchase Discounts ($5,200 x 4%)........................... 208
Cash ($5,200 x 96%)................................................ 4,992

Ex. 160
The following table summarizes the sales for the month of July for Perfect Platters Wholesalers Inc. The table
includes the terms, sales returns and when payment was collected for each sale.
Copyright © 2017 John Wiley & Sons Canada, Ltd. Unauthorized copying, distribution, or transmission of this page is prohibited
5 - 39 Merchandising Operations

Date Sale Amount Terms Returns Date Collected


April 3 $ 900 2/10, n/30 $ 50 April 9
April 5 1,300 3/10, n/30 200 April 21
April 11 450 1/10, n/30 0 April 13
April 18 2,300 4/10, n/60 520 April 25
April 22 1,600 2/10, n/30 750 May 5

Instructions
Calculate the cash received from each sale. Show your calculations.

Solution 160 (10 min.)


Apr 3 $ 833 ($900 – $50 = $850; $850 x 2% = $17; $850 – $17 = $833)

Apr 5 $ 1,100 ($1,300 – $200 = $1,100; discount not taken)

Apr 11 $ 445.50 ($450 x 1% = $4.50; $450 – $4.50 = $445.50)

Apr 18 $ 1,708.80 ($2,300 – $520 = $1,780; $1,780 x 4% = $71.20; $1,780 – $71.20 = $1,708.80)

Apr 22 $ 850 ($1,600 – $750 = $850; discount not taken)

Ex. 161
Storm Inc. completed the following transactions in October:
Credit Sales Sales Returns Date of
Date Amount Terms Date Amount Collection
Oct 3 $ 800 2/10, n/30 Oct 8
11 1,200 3/10, n/30 Oct 14 $ 500 16
17 7,000 1/10, n/30 20 1,200 29
21 1,700 2/10, n/60 23 400 27
23 2,500 2/10, n/30 27 500 28

Storm uses a perpetual inventory system.

Instructions
(a) Calculate the cash received for each collection. Show your calculations.
(b) Prepare the journal entry for the
(1) Oct. 17 sale. The merchandise sold had a cost of $3,500.
(2) Oct. 23 sales return. The merchandise returned had a cost of $200 and was returned to inventory.
(3) Oct. 28 collection.

Solution 161 (20 min.)


(a)
Oct 8 $784 [Sales $800 – Sales discount $16 ($800  2%)]

16 $679 [Sales $1,200 – Sales return $500 = $700;


$700 – Sales discount $21 ($700  3%)]

Copyright © 2017 John Wiley & Sons Canada, Ltd. Unauthorized copying, distribution, or transmission of this page is prohibited
Test
5 -Bank
40 for Financial Accounting: Tools for Business Decision-Making, Seventh Canadian Edition

29 $5,800 [Sales $7,000 – Sales return $1,200 = $5,800;


(discount not taken)]

27 $1,274 [Sales $1,700 – Sales return $400 = $1,300;


$1,300 – Sales discount $26 ($1,300  2%)]

28 $1,960 [Sales $2,500 – Sales return $500 = $2,000;


$2,000 – Sales discount $40 ($2,000  2%)]

(b)
(1) Oct 17 Accounts Receivable..................................................... 7,000
Sales....................................................................... 7,000
Cost of Goods Sold........................................................ 3,500
Inventory................................................................. 3,500

(2) 23 Sales Returns and Allowances...................................... 400


Accounts Receivable.............................................. 400
Inventory........................................................................ 200
Cost of Goods Sold................................................. 200

(3) 28 Cash.............................................................................. 1,960


Sales Discounts............................................................. 40
Accounts Receivable.............................................. 2,000

Ex. 162
Financial information is presented here for two companies. Complete the missing amounts.
Empty Corporation Full Corporation
Cost of goods sold $26,000 $?
Gross profit ? 38,000
Income tax expense 6,500 9,000
Net sales 47,000 62,000
Operating expenses 8,000 ?
Net income ? 9,000
Income before income tax 13,000 18,000
Sales 50,000 ?
Sales returns ? 5,000

Solution 162 (15 min.)


Empty Corporation
Sales returns = $3,000($50,000 – $47,000 = $3,000)
Gross profit = $21,000($47,000 – $26,000 = $21,000)
Net income = $6,500($21,000 – $8,000 – $6,500 = $6,500)

Full Corporation
Sales = $67,000($62,000 + $5,000 = $67,000)
Cost of goods sold = $24,000($62,000 – $38,000 = $24,000)
Operating expenses = $20,000($38,000 – $18,000 = $20,000)

Copyright © 2017 John Wiley & Sons Canada, Ltd. Unauthorized copying, distribution, or transmission of this page is prohibited
5 - 41 Merchandising Operations

Ex. 163
State the missing items identified by?.
(a) Gross profit – Operating expenses =?
(b) Sales – (? +?) = Net sales
(c) Income from operations +? –? = Income before income tax
(d) Net sales – Cost of goods sold =?
(e) Cost of goods sold + Gross profit =?

Solution 163 (5 min.)


(a) Income from operations

(b) Sales discounts, Sales returns and allowances

(c) Other revenues and gains, Other expenses and losses

(d) Gross profit

(e) Net sales

Ex. 164
The following information was taken from the adjusted trial balance of Lucifer Lighting Inc. at December 31,
2018. All accounts have normal balances.
Accounts payable............................................... $ 52,000
Accounts receivable........................................... 18,700
Accumulated depreciation—Building.................. 44,900
Advertising expense........................................... 38,500
Building.............................................................. 600,000
Cash................................................................... 85,000
Common shares................................................. 417,500
Cost of goods sold.............................................. 410,500
Depreciation expense......................................... 12,000
Freight out.......................................................... 22,000
Interest expense................................................. 5,700
Interest revenue................................................. 2,000
Rental revenue................................................... 6,000
Retained earnings, Jan 1................................... 154,800
Salaries expense................................................ 279,500
Salaries payable................................................. 5,200
Sales ................................................................. 798,500
Sales discounts ................................................. 8,200
Sales returns and allowances ............................ 29,000
Utilities expense . 9,200

Instructions
Copyright © 2017 John Wiley & Sons Canada, Ltd. Unauthorized copying, distribution, or transmission of this page is prohibited
Test
5 -Bank
42 for Financial Accounting: Tools for Business Decision-Making, Seventh Canadian Edition

Use the above information to prepare a multiple-step income statement for the year ended December 31,
2018.

Solution 164
LUCIFER LIGHTING INC.
Income Statement
Year Ended December 31, 2018
___________________________________________________________________________
Sales.................................................................................................. $798,500
Less: Sales returns and allowances................................................. $ 29,000
Sales discounts....................................................................... 8,200 37,200
Net sales............................................................................................ 761,300
Cost of goods sold.............................................................................. 410,500
Gross profit......................................................................................... 350,800
Operating expenses
Salaries expense......................................................................... $279,500
Advertising expense.................................................................... 38,500
Freight out................................................................................... 22,000
Depreciation expense.................................................................. 12,000
Utilities expense.......................................................................... 9,200
Total operating expenses..................................................... 361,200
Loss from operations.......................................................................... (10,400)
Other revenues and gains
Interest revenue.......................................................................... $ 2,000
Rental revenue............................................................................ 6,000
Other expenses and losses
Interest expense.......................................................................... 5,700 2,300
Loss................................................................................................... $ (8,100)

Ex. 165
Financial information is presented here for two companies:
Company A Company B
Cost of goods sold........................... $385,000 $?
Gross profit...................................... 395,000 438,000
Income tax expense......................... 38,000 ?
Net sales.......................................... 780,000 923,000
Operating expenses......................... ? 190,000
Net income....................................... ? 198,400
Income before income tax................ 190,000 248,000
Sales................................................ ? 950,000
Sales discounts................................ 6,000 ?
Sales returns and allowances.......... 14,000 18,000

Instructions
(a) Calculate the missing amounts for each company.
(b) For each company, calculate the gross profit margin and the profit margin.
(c) Which company is more profitable?

Copyright © 2017 John Wiley & Sons Canada, Ltd. Unauthorized copying, distribution, or transmission of this page is prohibited
5 - 43 Merchandising Operations

Solution 165 (20 min.)


(a) Company A
Sales $800,000 ($780,000 + $6,000 + $14,000)
Operating expenses $205,000 ($395,000 – $190,000)
Net income $152,000 ($190,000 – $38,000)

Company B
Sales discounts $9,000 ($950,000 – $18,000 – $923,000)
Cost of goods sold $485,000 ($923,000 – $438,000)
Income tax expense $49,600 ($248,000 – $198,400)

(b) Company A
Gross profit margin = 50.6% ($395,000 ÷ $780,000)
Profit margin = 19.5% ($152,000 ÷ $780,000)

Company B
Gross profit margin = 47.5% ($438,000 ÷ $923,000)
Profit margin = 21.5% ($198,400 ÷ $923,000)

(c) Although Company A has a higher gross profit margin, Company B is more profitable.

Ex. 166
The following information is available for Shawson Ltd. for calendar 2018:
Cost of goods sold....................................................... 595,000
Income tax expense.................................................... 4,500
Interest expense.......................................................... 15,000
Interest revenue.......................................................... 19,000
Operating expenses.................................................... 97,000
Sales........................................................................... $725,000
Sales returns and allowances...................................... 22,000

Instructions
(a) Use the above information to prepare a multiple-step income statement for the year ended December 31,
2018.
(b) Calculate the gross profit margin and the profit margin for 2018.

Solution 166 (20 min.)


(a) SHAWSON LTD.
Income Statement
Year Ended December 31, 2018
___________________________________________________________________________
Sales revenue
Sales.............................................................................................. $725,000
Less: Sales returns and allowances............................................... 22,000
Net sales........................................................................................ 703,000
Cost of goods sold......................................................................... 595,000
Gross profit.................................................................................... 108,000
Operating expenses....................................................................... 97,000
Copyright © 2017 John Wiley & Sons Canada, Ltd. Unauthorized copying, distribution, or transmission of this page is prohibited
Test
5 -Bank
44 for Financial Accounting: Tools for Business Decision-Making, Seventh Canadian Edition

Income from operations................................................................. 11,000


Other revenues and gains
Interest revenue...................................................................... $19,000
Other expenses and losses
Interest expense..................................................................... 15,000 (4,000)
Income before income tax.............................................................. 15,000
Income tax expense....................................................................... 4,500
Net income.................................................................................... $ 10,500

(b) Gross profit margin: $108,000 ÷ $703,000 = 15.4%


Profit margin: $10,500  $703,000 = 1.5%

Ex. 167
The adjusted trial balance of Sandhu Corporation at December 31, 2018 included the following selected
accounts:
Debit Credit
Advertising expense........................................... $ 15,000
Cost of goods sold.............................................. 347,000
Depreciation expense......................................... 3,296
Freight out.......................................................... 2,000
Income tax expense........................................... 32,000
Interest expense................................................. 19,000
Interest revenue................................................. $ 15,000
Sales.................................................................. 575,000
Sales discounts.................................................. 10,500
Sales returns and allowances............................. 55,000
Salaries expense................................................ 45,000
Utilities expense................................................. 18,000

Instructions
(a) Use the above information to prepare a multiple-step income statement for the year ended December 31,
2018.
(b) Calculate the gross profit margin and the profit margin for 2018.

Solution 167 (25 min.)


(a)
SANDHU CORPORATION
Income Statement
Year Ended December 31, 2018
___________________________________________________________________________
Sales.................................................................................................... $575,000
Less: Sales returns and allowances................................................... $55,000
Sales discounts......................................................................... 10,500 65,500
Net sales.............................................................................................. 509,500
Cost of goods sold................................................................................ 347,000
Gross profit........................................................................................... 162,500
Operating expenses
Copyright © 2017 John Wiley & Sons Canada, Ltd. Unauthorized copying, distribution, or transmission of this page is prohibited
5 - 45 Merchandising Operations

Salaries expense........................................................................... $45,000


Utilities expense............................................................................. 18,000
Advertising expense...................................................................... 15,000
Depreciation expense.................................................................... 3,296
Freight out..................................................................................... 2,000
Total operating expenses........................................................ 83,296
Income from operations........................................................................ 79,204
Other revenues and gains
Interest revenue............................................................................. $15,000
Other expenses and losses
Interest expense............................................................................ 19,000 4,000
Income before income tax.................................................................... 75,204
Income tax expense............................................................................. 32,000
Net income........................................................................................... $ 43,204

(b) Gross profit margin = $162,500 ÷ $509,500 = 31.9%


Profit margin = $43,204 ÷ $509,500 = 8.5%

Ex. 168
The adjusted trial balance of Jayco Corporation at December 31, 2018 included the following selected
accounts:
Debit Credit
Advertising expense........................................... $ 45,000
Cost of goods sold.............................................. 592,000
Depreciation expense......................................... 4,200
Freight out.......................................................... 11,200
Income tax expense........................................... 74,280
Interest expense................................................. 12,500
Interest revenue................................................. $ 15,000
Salaries expense................................................ 248,000
Sales.................................................................. 1,200,000
Sales discounts.................................................. 8,000
Sales returns and allowances............................. 34,000
Utilities expense................................................. 12,500

Instructions
(a) Use the above information to prepare a multiple-step income statement for the year ended December 31,
2018.
(b) Calculate the gross profit margin and the profit margin for 2018.

Solution 168 (25 min.)


(a)
JAYCO CORPORATION
Income Statement
Year Ended December 31, 2018
___________________________________________________________________________
Sales.................................................................................................. $1,200,000
Copyright © 2017 John Wiley & Sons Canada, Ltd. Unauthorized copying, distribution, or transmission of this page is prohibited
Test
5 -Bank
46 for Financial Accounting: Tools for Business Decision-Making, Seventh Canadian Edition

Less: Sales returns and allowances................................................. $ 34,000


Sales discounts....................................................................... 8,000 42,000
Net sales............................................................................................ 1,158,000
Cost of goods sold.............................................................................. 592,000
Gross profit......................................................................................... 566,000
Operating expenses
Salaries expense......................................................................... $248,000
Advertising expense.................................................................... 45,000
Utilities expense.......................................................................... 12,500
Freight out................................................................................... 11,200
Depreciation expense.................................................................. 4,200
Total operating expenses..................................................... 320,900
Income from operations...................................................................... 245,100
Other revenues and gains
Interest revenue.......................................................................... $ 15,000
Other expenses and losses
Interest expense.......................................................................... 12,500 2, 500
Income before income tax.................................................................. 247,600
Income tax expense........................................................................... 74,280
Net income......................................................................................... $ 173,320

(b) Gross profit margin = $566,000 ÷ $1,158,000 = 48.9%


Profit margin = $173,320 ÷ $1,158,000 = 15.0%

Ex. 169
Financial information is presented here for two companies.

Arts Inc. Cass Inc.


Sales $960,000 $ (e)
Sales returns and allowances 24,000 18,000
Sales discounts (a) 12,000
Net sales 920,000 834,000
Cost of goods sold 632,000 (f)
Gross profit 288,000 250,200
Operating expenses (b) 214,600
Income from operations 59,600 (g)
Other revenue (c) 4,300
Other expenses 3,200 1,800
Income before income tax 63,200 (h)
Income tax (d) 7,400
Net income 50,600 30,700

Instructions
(a) Fill in the missing amounts.
(b) Calculate the profit margin and the gross profit margin for each company.

Solution 169
(a)

Copyright © 2017 John Wiley & Sons Canada, Ltd. Unauthorized copying, distribution, or transmission of this page is prohibited
5 - 47 Merchandising Operations

Arts Inc. Cass Inc.


Sales $960,000 $864,000
Sales returns and allowances 24,000 18,000
Sales discounts 16,000 12,000
Net sales 920,000 834,000
Cost of goods sold 632,000 583,800
Gross profit 288,000 250,200
Operating expenses 228,400 214,600
Income from operations 59,600 35,600
Other revenue 6,800 4,300
Other expenses 3,200 1,800
Income before income tax 63,200 38,100
Income tax 12,600 7,400
Net income 50,600 30,700

(b) Gross profit margin Arts Inc. : ($288,000) / $920,000) = 31.3%


Cass Inc.: ($250,200) / $834,000) = 30.0%

Profit margin Arts Inc. : ($50,600) / $920,000) = 5.5%


Cass Inc.: ($30,700) / $834,000) = 3.7%

Ex. 170
Selected information from Coleman Inc.’s income statements for three years is indicated below:

2018 2017 2016


Net sales $225,800 $220,900 $219,696
Cost of goods sold 126,765 125,489 123,775
Income from operations 1,200 (7,495) 92
Net income 1,050 (7,750) 840

Instructions
(a) Calculate the gross profit margin for each year and comment on any trend in the percentages.
(b) Calculate the profit margin for each year and comment on any trend in the percentages.

Solution 170
(a) Gross profit margin 2018: [($225,800 – $126,765) / $225,800] = 43.9%
2017: [($220,900 – $125,489) / $220,900] = 43.2%
2016: [($219,696 – $123,775) / $219,696] = 43.7%

While there was a slight decrease from 2016 to 2017, the gross profit margin has remained relatively constant
over the three years.

(b) Profit margin 2018: ($1,050 / $225,800) = 0.5%


2017: [($7,750) / $220,900] = (3.5)%
2016: ($840 / $219,696) = 0.4%
Copyright © 2017 John Wiley & Sons Canada, Ltd. Unauthorized copying, distribution, or transmission of this page is prohibited
Test
5 -Bank
48 for Financial Accounting: Tools for Business Decision-Making, Seventh Canadian Edition

While there was a significant loss in 2017 the company more than recovered exceeding the profit margin
realized in 2016.

Ex. 171
The following information is available from recent financial statements of Competitor A and Competitor B:
(Amounts in millions)
Competitor A Competitor B
Cost of goods sold.................................... $21,761 $27,257
Income tax expense.................................. 361 766
Net sales................................................... 29,656 36,704
Operating expenses.................................. 7,962 10,435
Net income................................................ 594 1,072
Income before income tax......................... 955 1,838

Instructions
(a) Calculate the profit margin and gross profit margin for each company.
(b) What conclusions can be drawn from the ratios calculated in part (a) about the relative profitability of the
two companies?

Solution 171 (15 min.)


(a)
Competitor A Competitor B
Profit margin: $594 $1,072
———— = 2.0% ———— = 2.9%
$29,656 $36,704

Gross profit margin: ($29,656 – $21,761) ($36,704 – $27,257)


————————– ————————–
$29,656 $36,704

$7,895 $9,447
———— = 26.6% ———— = 25.7%
$29,656 $36,704

(b) Competitor B’s profit margin was 45% higher [(2.9% – 2.0%) ÷ 2.0%] than Competitor A’s, but Competitor
A’s gross profit margin was 3.5% higher [(26.6% – 25.7%) ÷ 25.7%] than Competitor B’s margin. It can be
concluded that Competitor B was slightly more profitable than Competitor A because its profit margin was
higher.

*Ex. 172
The following selected information for the year ended October 31, 2018 is for Trotman Inc.:

Advertising expense 81,500 Purchases returns and allowances 40,000


Freight In 20,000 Rent expense 24,000
Freight Out 26,000 Salaries expense 285,500
Income tax expense 52,620 Sales 1,476,000
Copyright © 2017 John Wiley & Sons Canada, Ltd. Unauthorized copying, distribution, or transmission of this page is prohibited
5 - 49 Merchandising Operations

Inventory, November 1, 2017 90,000 Sales discounts 8,000


Inventory, October 31, 2018 51,600 Sales returns and allowances 56,000
Purchases 800,000 Utilities expense 13,200
Purchase discounts 12,000

Instructions
(a) Prepare a multiple-step income statement for Trotman Inc. for the year ended October 31, 2018.
(b) Calculate the gross profit margin and profit margin for the year.

*Solution 172
(a)
TROTMAN INC.
Income Statement
Year Ended October 31, 2018

Sales.................................................................................................... $1,476,000
Less: Sales returns and allowances................................................... $ 56,000
Sales discounts......................................................................... 8,000 64,000
Net sales.............................................................................................. 1,412,000
Cost of goods sold
Inventory, November 1, 2017............................................................... $ 90,000
Purchases............................................................................................ 800,000
Less: Purchases returns and allowances.........................$40,000
Purchase discounts................................................. 12,000 52,000
Net purchases...................................................................................... 748,000
Add: Freight in................................................................................... 20,000
Cost of goods purchased...................................................................... 768,000
Cost of goods available for sale............................................................ 858,000
Inventory, October 31, 2018................................................................. 51,600
Cost of goods sold................................................................................ 806,400
Gross profit........................................................................................... $605,600
Operating expenses
Salaries expense......................................................................... $285,500
Advertising expense.................................................................... 81,500
Freight out................................................................................... 26,000
Rent expense.............................................................................. 24,000
Utilities expense.......................................................................... 13,200
Total operating expenses..................................................... 430,200
Income before income tax.................................................. 175,400
Income tax expense........................................................... 52,620
Net income......................................................................... $ 122,780

(b) Gross profit margin = $605,600 ÷ $1,412,000 = 42.9%

Profit margin = $122,780 ÷ $1,412,000 = 8.7%

*Ex. 173
Summarized below are the transactions recorded by Rummy Ltd. for calendar 2018, using a perpetual
inventory system. Their Jan 1 opening balances were: accounts receivable $145,000, inventory $45,000, and
Copyright © 2017 John Wiley & Sons Canada, Ltd. Unauthorized copying, distribution, or transmission of this page is prohibited
Test
5 -Bank
50 for Financial Accounting: Tools for Business Decision-Making, Seventh Canadian Edition

accounts payable $122,000.

Inventory............................................................................................... 400,000
Accounts Payable.......................................................................... 400,000
(Purchase of inventory)

Inventory............................................................................................... 10,000
Cash.............................................................................................. 10,000
(Payment of freight in on inventory)

Accounts Payable................................................................................. 20,000


Inventory........................................................................................ 20,000
(Returned merchandise to supplier for credit)

Accounts Receivable............................................................................ 538,000


Cash..................................................................................................... 200,000
Sales.............................................................................................. 738,000
(Record sales for year)

Cost of Goods Sold.............................................................................. 420,000


Inventory........................................................................................ 420,000
(Record COGS for year)

Sales Returns and Allowances............................................................. 28,000


Accounts Receivable..................................................................... 28,000
(Record goods returned from customers)

Inventory............................................................................................... 16,800
Cost of Goods Sold........................................................................ 16,800
(Record goods returned from customers)

Accounts Payable................................................................................. 400,000


Inventory........................................................................................ 6,000
Cash.............................................................................................. 394,000
(Record payments to suppliers, with a $6,000 purchase discount)

Cash..................................................................................................... 560,000
Sales Discounts.................................................................................... 4,000
Accounts Receivable..................................................................... 564,000
(Record receipts from customers)

Instructions
Prepare the 2018 income statement to the gross profit line only.
(a) As it would appear using the perpetual inventory system.
(b) As it would appear if a periodic inventory system had been used.
(c) Calculate the gross profit margin for the year.

*Solution 173 (20–25 min.)


(a) Perpetual
RUMMY LTD.
Income Statement (partial)
Copyright © 2017 John Wiley & Sons Canada, Ltd. Unauthorized copying, distribution, or transmission of this page is prohibited
5 - 51 Merchandising Operations

Year Ended December 31, 2018


___________________________________________________________________________
Sales.................................................................................................... $738,000
Less: Sales returns and allowances..................................................... $28,000
Sales discounts.............................................................................. 4,000 32,000
Net sales.............................................................................................. 706,000
Cost of goods sold*.............................................................................. 403,200
Gross profit........................................................................................... $302,800

* $420,000 – $16,800 = $403,200

(b) Periodic
RUMMY LTD.
Income Statement (partial)
Year Ended December 31, 2018
Sales.................................................................................................... $738,000
Less: Sales returns and allowances................................................... $ 28,000
Sales discounts......................................................................... 4,000 32,000
Net sales.............................................................................................. 706,000
Cost of goods sold
Inventory, January 1............................................................................. $ 45,000
Purchases............................................................................................ 400,000
Less: Purchases returns and allowances.........................$20,000
Purchase discounts................................................. 6,000 26,000
Net purchases...................................................................................... 374,000
Add: Freight in................................................................................... 10,000
Cost of goods purchased...................................................................... 384,000
Cost of goods available for sale............................................................ 429,000
Inventory, December 31*...................................................................... 25,800
Cost of goods sold................................................................................ 403,200
Gross profit........................................................................................... $302,800

*Since cost of goods sold is the same as under the perpetual system, ending inventory must be
$429,000 – $403,200 = $25,800.

(c) Gross profit margin = $302,800 ÷ $706,000 = 42.9%

*Ex. 174
Below is a series of cost of goods sold sections for four companies that use a periodic inventory system (in
thousands):
Co. A Co. B Co. C Co. D
Beginning inventory (a) 35 12 (m)
Purchases 123 (e) 67 (n)
Purchase returns and allowances (b) 9 (i) 11
Net purchases 113 205 66 178
Freight in (c) 20 (j) 12
Freight out 10 12 9 8
Cost of goods purchased 147 (f) 73 (o)
Cost of goods available for sale 171 (g) (k) 190
Copyright © 2017 John Wiley & Sons Canada, Ltd. Unauthorized copying, distribution, or transmission of this page is prohibited
Test
5 -Bank
52 for Financial Accounting: Tools for Business Decision-Making, Seventh Canadian Edition

Ending inventory (d) (h) 8 (p)


Cost of goods sold 141 235 (l) 171

Instructions
What are the amounts that should appear in the table where a letter in parentheses is shown?

*Solution 174 (15–20 min.)


($ in thousands) Co. A Co. B Co. C Co. D
Beginning inventory $ 24 $ 35 $12 $ 0
Purchases 123 214 67 189
Purchase returns and allowances 10 9 1 11
Net purchases 113 205 66 178
Freight in 34 20 7 12
Freight out 10 12 9 8
Cost of goods purchased 147 225 73 190
Cost of goods available for sale 171 260 85 190
Ending inventory 30 25 8 19
Cost of goods sold 141 235 77 171

*Ex. 175
On June 1, Grill Master Ltd. had an inventory of 10 barbeques at a cost of $440 each. Grill Master uses a
periodic inventory system. During the month of June the following transactions occurred:
Jun 3 Purchased 25 barbeques at a cost of $440 each from BBQ King Ltd., terms n/30.
5 Paid $200 freight for the barbeques purchased on June 3.
6 Sold 12 barbeques to Outdoor Grill for $760 each, terms 2/10, n/30.
7 Received credit from BBQ King for the return of two defective barbeques.
13 Issued a credit to Outdoor Grill for the return of one defective barbeque.
16 Received a credit from BBQ King for the defective barbeque returned by Outdoor Grill.
19 Purchased 10 barbeques from Backyard Barbecues at a cost of $440 each, terms 2/10, n/30.
20 Paid freight of $200 on the June 19 purchase.

On June 30, Grill Masters ending inventory was $6,440

Instructions
(a) Prepare journal entries to record the above transactions.
(b) Calculate the cost of goods sold for June.

*Solution 175 (20 min.)


(a)
Jun 3 Purchases ($440 x 25)....................................................... 11,000
Accounts Payable........................................................ 11,000

5 Freight In............................................................................ 200


Cash............................................................................ 200

Copyright © 2017 John Wiley & Sons Canada, Ltd. Unauthorized copying, distribution, or transmission of this page is prohibited
5 - 53 Merchandising Operations

6 Accounts Receivable ($760 x 12)....................................... 9,120


Sales........................................................................... 9,120

7 Accounts Payable ($440 x 2).............................................. 880


Purchase Returns and Allowances.............................. 880

13 Sales Returns and Allowances........................................... 760


Accounts Receivable................................................... 760

16 Accounts Payable............................................................... 440


Purchase Returns and Allowances.............................. 440

19 Purchases ($440 x 10)....................................................... 4,400


Accounts Payable........................................................ 4,400

20 Freight In............................................................................ 200


Cash............................................................................ 200

(b)
Inventory, June 1 (10 @ $440)...................................................... $4,400
Purchases (35 @ $440)................................................................. $15,400
Less: purchase returns and allowances (3 @ $440)...................... 1,320
Net purchases............................................................................... 14,080
Add: freight in ($200 + $200)......................................................... 400
Cost of goods purchased............................................................... 14,480
Cost of goods available for sale..................................................... 18,880
Inventory, June 30......................................................................... 6,440
Cost of goods sold......................................................................... $12,440

*Ex. 176
Magnesium Inc. uses a periodic inventory system. During April, the following transactions occurred:
Apr 3 Purchased $2,000 of merchandise, terms 3/10, n/60.
6 Returned $300 of the merchandise purchased on April 3.
7 Paid freight charges of $150 on goods purchased on April 3.
12 Paid for the goods purchased on April 3.
13 Sold goods on credit for $1,000, terms 2/10, n/45.
14 The customer of April 13 returned $300 of the goods.
23 Received payment from the customer of April 13.

Instructions
Prepare journal entries to record the above transactions.

*Solution 176 (20 min.)


Apr 3 Purchases.......................................................................... 2,000
Accounts Payable........................................................ 2,000

6 Accounts Payable............................................................... 300


Copyright © 2017 John Wiley & Sons Canada, Ltd. Unauthorized copying, distribution, or transmission of this page is prohibited
Test
5 -Bank
54 for Financial Accounting: Tools for Business Decision-Making, Seventh Canadian Edition

Purchase Returns and Allowances.............................. 300

7 Freight In............................................................................ 150


Cash............................................................................ 150

12 Accounts Payable ($2,000 – $300)..................................... 1,700


Purchase Discounts ($1,700 x 3%)............................. 51
Cash ($1,700 x 97%)................................................... 1,649

13 Accounts Receivable.......................................................... 1,000


Sales........................................................................... 1,000

14 Sales Returns and Allowances........................................... 300


Accounts Receivable................................................... 300

23 Cash ($700 x 98%)............................................................. 686


Sales Discounts ($700 x 2%)............................................. 14
Accounts Receivable ($1,000 – $300)......................... 700

*Ex. 177
Pacific Supply Corporation uses a periodic inventory system. During September, the following transactions
occurred:
Sep 3 Purchased 36 backpacks at $25 each from Scott Limited, terms 2/10, n/30.
6 Received credit of $100 for the return of 4 backpacks purchased on Sept. 3 that were defective.
9 Sold 20 backpacks for $45 each to Macklin Books, terms 2/10, n/30.
13 Paid Scott account in full.

Instructions
Prepare journal entries to record the above transactions.

*Solution 177 (15 min.)


Sep 3 Purchases ($25 x 36)......................................................... 900
Accounts Payable........................................................ 900

6 Accounts Payable............................................................... 100


Purchase Returns and Allowances.............................. 100

9 Accounts Receivable ($45 x 20)......................................... 900


Sales........................................................................... 900

13 Accounts Payable ($900 – $100)....................................... 800


Purchase Discounts ($800 × 2%)................................ 16
Cash ($800 x 98%)...................................................... 784

*Ex. 178
Babylon Corporation uses a periodic inventory system. During October, the following transactions occurred:
Oct 3 Purchased $16,000 of merchandise on credit, terms 4/10, n/30.
6 Returned $1,600 of the goods purchased on Oct 3.
Copyright © 2017 John Wiley & Sons Canada, Ltd. Unauthorized copying, distribution, or transmission of this page is prohibited
5 - 55 Merchandising Operations

7 Paid freight charges of $250 for goods purchased on Oct 3.


12 Paid for the goods purchased on Oct 3.

Instructions
Prepare journal entries to record the above transactions.

*Solution 178 (15 min.)


Oct 3 Purchases.......................................................................... 16,000
Accounts Payable........................................................ 16,000

6 Accounts Payable............................................................... 1,600


Purchase Returns and Allowances.............................. 1,600

7 Freight In............................................................................ 250


Cash............................................................................ 250

12 Accounts Payable ($16,000 – $1,600)................................ 14,400


Purchase Discounts ($14,400x 4%)............................ 576
Cash ($14,400 x 96%)................................................. 13,824

*Ex. 179
The most recent income statement of Lawerence Limited includes the items listed below:
Beginning inventory..................................................... $ 900,000
Freight in..................................................................... 20,000
Gross profit.................................................................. 1,400,000
Net sales..................................................................... 3,750,000
Operating expenses.................................................... 300,000
Purchases................................................................... 1,520,000
Purchase discounts..................................................... 35,000
Purchase returns and allowances................................ 12,000

Instructions
Use the appropriate items listed above as a basis for calculating:
(a) Cost of goods sold.
(b) Cost of goods available for sale.
(c) Ending inventory.

*Solution 179 (15 min.)


(a) Net sales – Cost of goods sold = Gross profit
$3,750,000 – Cost of goods sold = $1,400,000
Cost of goods sold = $2,350,000

(b) Beginning inventory..................................................... $ 900,000


Purchases................................................................... $1,520,000
Less: Purchase discounts............................................ $35,000
Purchase returns and allowances...................... 12,000 47,000

Copyright © 2017 John Wiley & Sons Canada, Ltd. Unauthorized copying, distribution, or transmission of this page is prohibited
Test
5 -Bank
56 for Financial Accounting: Tools for Business Decision-Making, Seventh Canadian Edition

Net Purchases............................................................. 1,473,000


Add: Freight in............................................................ 20,000
Cost of goods purchased............................................. 1,493,000
Cost of goods available for sale................................... $2,393,000

(c) Cost of goods available for sale – Ending inventory = Cost of goods sold
$2,393,000 – Ending inventory = $2,350,000
Ending inventory = $43,000

*Ex. 180
Given the following information, prepare in good form the cost of goods sold section of an income statement,
using the periodic inventory system.
Beginning inventory..................................................... $30,000
Ending inventory.......................................................... 32,000
Freight in..................................................................... 8,000
Purchases................................................................... 76,000
Purchase discounts..................................................... 1,000
Purchase returns and allowances................................ 3,600

*Solution 180 (15 min.)


Beginning inventory............................................................ $30,000
Purchases.......................................................................... $76,000
Less: Purchase returns and allowances............................ $3,600
Purchase discounts.................................................. 1,000 4,600
Net purchases.................................................................... 71,400
Freight in............................................................................ 8,000
Cost of goods purchased.................................................... 79,400
Cost of goods available for sale......................................... 109,400
Ending inventory................................................................. 32,000
Cost of goods sold.............................................................. $77,400

*Ex. 181
Three items are missing in each of the following columns and are identified by a letter.
Sales $ (a) $1,720,000
Sales returns and allowances 30,000 40,000
Sales discounts 20,000 30,000
Net sales 900,000 (d)
Beginning inventory (b) 650,000
Cost of goods purchased 400,000 (e)
Ending inventory 340,000 606,000
Cost of goods sold 500,000 1,150,000
Gross profit (c) (f)

Copyright © 2017 John Wiley & Sons Canada, Ltd. Unauthorized copying, distribution, or transmission of this page is prohibited
5 - 57 Merchandising Operations

Instructions
Calculate the missing amounts and identify them by letter.

*Solution 181 (15 min.)


(a) $950,000

(b) $440,000

(c) $400,000

(d) $1,650,000

(e) $1,106,000

(f) $500,000

*Ex. 182
Mendez Electronics Limited uses the periodic inventory system and prepares monthly financial statements. All
accounts have been adjusted except for inventory. A physical count of inventory on September 30, 2018
indicates that $2,000 was on hand. A partial listing of adjusted account balances follows:
Accounts payable........................................................ $ 7,250
Accounts receivable.................................................... 8,000
Cash............................................................................ 22,000
Freight in..................................................................... 1,100
Income tax expense.................................................... 1,530
Inventory, September 1 .............................................. 1,500
Operating expenses.................................................... 23,100
Purchases................................................................... 35,000
Purchase returns and allowances................................ 350
Sales........................................................................... 70,000
Sales discounts........................................................... 750

Instructions
Prepare a multiple-step income statement for Mendez Electronics for the month ended September 30, 2018.

*Solution 182 (15 min.)


MENDEZ ELECTRONICS LIMITED
Income Statement
Month Ended September 30, 2018
___________________________________________________________________________
Sales revenue
Sales........................................................................... $70,000
Less: Sales discounts.................................................. 750
Net sales..................................................................... $69,250

Cost of goods sold


Inventory, September 1............................................... $ 1,500
Purchases................................................................... $35,000
Copyright © 2017 John Wiley & Sons Canada, Ltd. Unauthorized copying, distribution, or transmission of this page is prohibited
Test
5 -Bank
58 for Financial Accounting: Tools for Business Decision-Making, Seventh Canadian Edition

Less: Purchase returns and allowances...................... 350


Net purchases............................................................. 35,350
Add: Freight in............................................................. 1,100
Cost of goods purchased............................................. 36,450
Cost of goods available for sale................................... 37,950
Inventory, September 30............................................. 2,000
Cost of goods sold................................................ 35,950
Gross profit......................................................................... 33,300
Operating expenses........................................................... 23,100
Income before income tax.................................................. 10,200
Income tax expense........................................................... 1,530
Net income......................................................................... $ 8,670

Copyright © 2017 John Wiley & Sons Canada, Ltd. Unauthorized copying, distribution, or transmission of this page is prohibited
5 - 59 Merchandising Operations

MATCHING QUESTIONS

183. Match the items below by entering the appropriate code letter in the space provided.

A. Net sales F. Contra revenue


B. Sales discount G. Freight out
C. Credit terms H. Gross profit
D. Periodic inventory system I. Sales invoice
E. Gross profit margin J. Purchase discount

_____ 1. A reduction given by the seller for prompt payment of a credit sale

_____ 2. Provides support for a credit sale

_____ 3. Gross profit divided by net sales

_____ 4. Sales less sales returns and allowances and sales discounts

_____ 5. Specifies the amount of cash discount and time period during which it is offered

_____ 6. Net sales less cost of goods sold

_____ 7. Freight cost to deliver goods to customers reported as an operating expense

_____ 8. Requires a physical count of goods on hand to calculate cost of goods sold

_____ 9. A cash discount claimed by a buyer for prompt payment of a balance due

_____ 10. An account that is offset against a revenue account on the income statement

Copyright © 2017 John Wiley & Sons Canada, Ltd. Unauthorized copying, distribution, or transmission of this page is prohibited
Test
5 -Bank
60 for Financial Accounting: Tools for Business Decision-Making, Seventh Canadian Edition

ANSWERS TO MATCHING

1. B

2. I

3. E

4. A

5. C

6. H

7. G

8. D

9. J

10. F

Copyright © 2017 John Wiley & Sons Canada, Ltd. Unauthorized copying, distribution, or transmission of this page is prohibited
5 - 61 Merchandising Operations

SHORT-ANSWER ESSAY QUESTIONS

S-A E 184
There are different types of inventories and physical assets that an organization may report on their statement
of financial position.
(a) What kind of businesses would report what type of inventory?
(b) Explain the difference between inventory and property, plant and equipment.

Solution 184
(a) Retailers and wholesalers would report inventory, which is in a form ready to sell to customers (e.g.,
Walmart, Loblaw, etc.).
Manufacturers would report raw materials inventory (basic materials on hand ready to go into
production), work in process inventory (inventory that has been started into production but is not yet
complete), and finished goods inventory (manufactured items that are complete and ready for sale).

(b) Inventory has two common characteristics: (1) the company owns it, and (2) it is either in the process of
production for sale to customers or in the form ready for sale to customers.
Property, plant, and equipment are tangible assets with relatively long useful lives that are currently being
used in operating the business. This category includes items such as land, buildings, equipment, furniture,
computers, and vehicles.

S-A E 185
The periodic and the perpetual inventory systems are two methods that companies use to account for
inventories. Briefly describe the major features of each system and explain why a physical inventory is
necessary under both systems.

Solution 185
When a periodic inventory system is used, the Inventory account remains the same throughout the period.
Separate accounts, such as Purchases, Freight In, and Purchase Discounts, are used to record the
transactions. Cost of goods sold is determined by the following formula:
Beginning inventory + Purchases – Ending inventory.

The determination of ending inventory is made by a physical count.

When a perpetual inventory system is used, the purchase and sale of goods are recorded directly in the
Inventory account, which eliminates the need for separate accounts. Cost of goods sold is recognized for each
sale by debiting Cost of Good Sold and crediting Inventory. At the end of the period, the ending account
balance should equal inventory's ending balance. However, a company should conduct a physical inventory
count at least once a year, because there could be differences resulting from spoilage, theft, or errors.

S-A E 186
What is the main consideration when choosing between a periodic and a perpetual inventory system?

Solution 186
When choosing between a periodic and perpetual inventory system, a company should consider the additional
costs associated with keeping detailed inventory records versus the benefits of having additional information
about, and control over their inventory.

Copyright © 2017 John Wiley & Sons Canada, Ltd. Unauthorized copying, distribution, or transmission of this page is prohibited
Test
5 -Bank
62 for Financial Accounting: Tools for Business Decision-Making, Seventh Canadian Edition

S-A E 187
Distinguish between cost of goods sold, operating expenses, and non-operating expenses. Describe the nature
of these three items and their placement on a multiple-step income statement.

Solution 187
Cost of goods sold includes the cost of obtaining the goods held for resale; it is deducted directly from net
sales on the income statement. Net sales less cost of goods sold results in gross profit. Operating expenses,
on the other hand, appear directly below the gross profit on the income statement. Operating expenses include
the costs of running the day-to-day operations of the business such as rent, salaries and insurance. Non-
operating expenses are expenses unrelated to daily operations, such as interest expense.

S-A E 188
The income statement for a merchandising company presents three amounts not shown in a service
company’s income statement. The statement of financial position for a manufacturing company presents
different inventory accounts not shown in a merchandising company’s statement of financial position. Identify
and briefly explain the differences for each.

Solution 188
The items reported on an income statement for a merchandising company that are not reported for a service
company include: net sales revenues, cost of goods sold, and gross profit. Net sales revenues consist of sales,
sales returns and allowances, and sales discounts. Cost of goods sold represents the total cost of
merchandise sold during the period. Gross profit is the excess of net sales over the cost of goods sold.

The inventory items reported on a statement of financial position for a manufacturing company that are not
reported for a merchandising company include: raw materials, work in process and finished goods inventory.
Raw materials reflect basic materials on hand ready to go into production, work in process inventory reflects
inventory that has been started in production but is not yet complete and finished goods inventory reflects
manufactured items that are complete and ready for sale.

S-A E 189
Public companies in Canada must list expenses on the income statement either by nature or function. Explain
what this means. Are private companies required to do the same?

Solution 189
Classifying expenses by nature means that expenses are reported according to their natural classifications,
e.g., salaries, depreciation, advertising, utilities. Classifying expenses by function means that expenses are
reported according to the activity (business function) for which they were incurred, e.g., cost of goods sold,
administration, selling expenses. An organization has the choice to classify by nature or by function – the
choice should be based on whichever provides more relevant information. Note expenses may be listed in any
order within the chosen classification. Note also that organizations following ASPE may list their expenses in
whatever order they choose, or they may list by nature or function.

S-A E 190
In a single-step income statement, all data (except for income tax) are classified under two categories: (1)
Revenues, or (2) Expenses. If the income statement is recast in a multiple-step format, what additional
information or intermediate components of revenue would be presented?

Solution 190
Copyright © 2017 John Wiley & Sons Canada, Ltd. Unauthorized copying, distribution, or transmission of this page is prohibited
5 - 63 Merchandising Operations

The items reported in a multiple-step income statement that are not reported in a single-step income statement
are gross revenues as well as net revenues, cost of goods sold, gross profit, operating expenses, income from
operations, other revenues and gains, and other expenses and losses.

S-A E 191
You are working for the summer at PLC Ltd., a company that operates a chain of retail stores. In past, the
company has not disclosed its cost of goods sold, but now is required to do so. The company president would
like to know the pros and cons of disclosing information. Prepare a memo to the president containing the
information requested.

Solution 191
MEMO
TO: President, PLC Ltd.
FROM: Accounting Student
RE: Disclosure of cost of goods sold
DATE: June xx, xxxx

Disclosing the cost of goods sold enables users of the statements to better evaluate the
company’s performance. They can see the relationship between the company’s sales and its
cost of goods sold. The downside of disclosing the information is that competitors can also have
access to this information. For example, they can use the information to estimate the company’s
markup although its value will be limited as they can only calculate the markup in its aggregate
(total) and not by product category.

S-A E 192
You are working as an accounting clerk for Jakubo Wholesalers for the summer. You notice that some invoices
that look like inventory purchases are debited to the Operating Expenses account. When you ask your
supervisor about the invoices, she says you don’t need to be concerned about it because it won’t have any
effect on the net income.

Instructions
Does the classification of the invoices matter? Explain.

Solution 192
Classifying the invoices as operating expenses rather than inventory will have the immediate effect of
understating current assets on the statement of financial position and cost of goods sold on the income
statement. Subsequently, when the inventory is sold in a later period, cost of goods sold will be understated
and gross profit overstated.

Not properly distinguishing on the income statement between cost of goods sold and operating expenses will
increase the gross profit and gross profit margin. The gross margin is important in evaluating the company’s
performance, so the misclassification does matter.

S-A E 193
Explain why gross profit margin is considered to be more informative than gross profit.

Solution 193
Copyright © 2017 John Wiley & Sons Canada, Ltd. Unauthorized copying, distribution, or transmission of this page is prohibited
Test
5 -Bank
64 for Financial Accounting: Tools for Business Decision-Making, Seventh Canadian Edition

Gross profit margin expresses a more meaningful relationship between gross profit and sales. Specifically, it
shows how much gross profit a company earns for each $1 in net sales it generates. This puts gross profit into
perspective and draws attention to a company’s profitability relative to its size.

*S-A E 194
A merchandising company using the periodic system frequently has the need to use contra accounts related to
the purchase and sale of goods. Identify the contra accounts that have (1) normal credit balances and explain
why they are not considered revenues, and (2) normal debit balances and explain why they are not considered
expenses.

*Solution 194
1. The contra accounts related to the purchase of goods that have normal credit balances are Purchase
Discounts and Purchase Returns and Allowances. These accounts have credit balances because they are
adjustments to purchases, not revenues. They are an adjustment of the outflow from the purchase of
goods, rather than a revenue generating activity.

2. The contra accounts related to the sale of goods that have normal debit balances are Sales Discounts and
Sales Returns and Allowances. These accounts have debit balances but are not expenses because they
are adjustments of sales, not operating, selling, or administrative expenses. They are an adjustment of the
inflow from sale of goods, rather than a cost used to help earn revenue.

Copyright © 2017 John Wiley & Sons Canada, Ltd. Unauthorized copying, distribution, or transmission of this page is prohibited
5 - 65 Merchandising Operations

OBJECTIVE FORMAT QUESTIONS

195. The following is a selection of journal entries that Glipon Corporation recorded in July 2018. Glipon uses a
perpetual inventory system. Indicate all of the entries that were recorded correctly.

(a) July 31 Cash 9,800


Sales Discount 200
Accounts Receivable 10,000
(To record receipt of payment on account on sales to TGT
Company. TGT paid within the discount period (terms 2/10,
n/30))

(b) July 2 Inventory 4,900


Accounts Payable 4,900
(To record goods purchased on account, terms 2/10, n/30,)

(c) July 2 Freight In 115


Cash 115
(To record payment of freight on goods purchased on July 2,
FOB shipping point)

(d) July 8 Accounts Payable 400


Inventory 400
[To record the return of a portion of the goods purchased on
account on July 2; see transaction (b)]

(e) July 11 Accounts Payable 4,500


Cash 4,410
Purchase Discount 90
[To record payment for remainder of goods purchased on
July 2; see transaction (b)]

(f) July 15 Accounts Payable 1,500


Cash 1,500
(To record payment for goods purchased in June—no
discount taken)

(g) July 31 Freight Out 185


Cash 185
(To record freight costs on sales to customer shipped FOB
shipping point)

(h) July 31 Sales 400


Accounts Receivable 400
(To record return of goods sold to RBB Company on account)

Solution 195
(a), (b), (d), and (f) are correct; (c), (e), (g), and (h) are not correct.
Copyright © 2017 John Wiley & Sons Canada, Ltd. Unauthorized copying, distribution, or transmission of this page is prohibited
Test
5 -Bank
66 for Financial Accounting: Tools for Business Decision-Making, Seventh Canadian Edition

(c) When freight is recorded on goods purchased in a perpetual inventory system, the cost of the freight is
added to the inventory value for the goods. The journal entry should be:
Inventory 115
Cash 115

(e) Under a perpetual inventory system, when a discount is taken on a purchase, the value of inventory
reported by the company is decreased. The journal entry should be:
Accounts Payable 4,500
Cash 4,410
Inventory 90

(g) If Glipon shipped the goods FOB shipping point, the shipping costs are the responsibility of the customer,
not Glipon. Therefore, this entry should not be made and the shipping costs should be paid by the
customer.

(h) When goods sold are returned, the contra account, Sales Returns and Allowances, is debited. This
account is used, instead of debiting Sales, so management can monitor returns. The journal entry should
be:
Sales Returns and Allowances 400
Accounts Receivable 400

A corresponding entry for amount of the cost of goods returned to inventory would be recorded as follows:
Inventory XXX’
Cost of Goods Sold XXX

196. Indicate which of the following statements about the income statement are correct:
(a) Companies that report under ASPE should report expenses in a specified order on the income statement.
(b) Companies that report under IFRS must classify expenses by nature or function; whichever is more
relevant.
(c) A company that reports under IFRS and classifies expenses by nature must disclose additional information
on the function of certain expenses, such as whether they are related to administration, production or
sales.
(d) More management judgement is required if a company decides to classify expenses by function.
(e) Most Canadian companies use the single-step form of income statement.
(f) Income before income tax is only reported on the multiple-step form of income statement.
(g) On a multiple-step income statement, income from operations is determined by subtracting operating
expenses from gross profit.
(h) For evaluative purposes, income from operations has more predictive value than income before tax.

Solution 196
(b), (d), (g), and (h) are correct; while (a), (c), (e), and (f) are incorrect.

(a) Companies that report under ASPE do not need to report expenses in any particular order on the income
statement. Expenses can be classified in any manner the company finds useful.

(c) A company that reports under IFRS and classifies expenses by nature does not need to disclose any
additional information about the function of expenses. On the other hand, a company that reports under
IFRS and classifies expenses by function, they must disclose additional information on the nature of the
Copyright © 2017 John Wiley & Sons Canada, Ltd. Unauthorized copying, distribution, or transmission of this page is prohibited
5 - 67 Merchandising Operations

expenses. If, for instance, expenses are shown on the income statement as administrative expenses, the
breakdown of these expenses would be disclosed in the notes.

(e) Most Canadian companies use the multiple-step format of income statement. The multiple-step income
statement provides more detail by segregating operating revenues and expenses from non-operating
revenues and expenses.

(f) Income before tax is reported on both the single-step and multiple-step income statements.

197. The following incomplete multiple-step income statement is provided for Almond Industries Ltd. for the
year ended November 30, 2018.

Almond Industries Ltd.


Income Statement
Year Ended November 30, 2018

Sales ........................................................................................................... $409,000


Less: Sales returns and allowances................................ $42,000
Sales discounts..................................................... a. b.
Net sales.............................................................................................. c.
Cost of goods sold ........................................................................................ d.
Gross profit ................................................................................................... 186,955
Operating expenses
Salaries expense ............................................................. $47,500
Depreciation expense....................................................... 4,500
Freight-out ..................................................................... 1,200
Insurance expense .......................................................... 1,800
Total operating expenses............................................................ e.
Income from operations................................................................................. 131,955
Other revenues
Interest revenue................................................................................... f.
Income before income tax.............................................................................. g.
Income tax expense....................................................................................... h.
Net income..................................................................................................... $104,941

Additional Information: The income tax rate is 11%. Of the $409,000 in sales, 25% of the customers took the
sales discount (terms offered to all customers are 2/10, n/30).

Complete the missing data in the income statement by choosing the corresponding number for each missing
item from the following list. Note: some values will not be used.

Copyright © 2017 John Wiley & Sons Canada, Ltd. Unauthorized copying, distribution, or transmission of this page is prohibited
Test
5 -Bank
68 for Financial Accounting: Tools for Business Decision-Making, Seventh Canadian Edition

a.    Sales discounts  


b.    Total of Sales returns and allowances and Sales discounts  
c.    Net sales  
d.    Cost of goods sold  
e.    Total operating expenses  
f.     Interest revenue  
g.    Income before income tax  
h.    Income tax expense  

Solution 197

a. Sales discounts
Sales discounts $409,000 x 25% x 2% = $2,045 6

b. Total of Sales returns and allowances and Sales discounts


Sales discounts $2,045 from (a) + Sales returns and
Allowances of $42,000 = $44,045 2

c. Net sales
Sales less (b) = $409,000 – $44,045 = $364,955 1

d. Cost of goods sold 4

Net sales (c)$ 364,955 less gross profit of $186,955 = $178,000


e. Total operating expenses 17

$47,500 + $4,500 + $1,200 + $1,800 = $55,000


f. Interest revenue
Income before income taxes -Income from operations
$132,875 - $131,955 = $920 9

g. Income before income tax


Net income ÷ (1-.21) = $132,875 15

h. Income tax expense


Income before income tax $132,875 x 21% = $27,904 10
Copyright © 2017 John Wiley & Sons Canada, Ltd. Unauthorized copying, distribution, or transmission of this page is prohibited
5 - 69 Merchandising Operations

The completed multiple-step income statement for Almond Industries Ltd. is as follows:

Almond Industries Ltd.


Income Statement
Year Ended November 30, 2018

Sales ........................................................................................................... $409,000


Less: Sales returns and allowances................................ $42,000
Sales discounts..................................................... 2,045 44,045
Net sales.............................................................................................. 364,955
Cost of goods sold ........................................................................................ 178,000
Gross profit ................................................................................................... 186,955
Operating expenses
Salaries expense ............................................................. $47,500
Depreciation expense....................................................... 4,500
Freight-out ..................................................................... 1,200
Insurance expense .......................................................... 1,800
Total operating expenses............................................................ 55,000
Income from operations................................................................................. 131,955
Other revenues
Interest revenue................................................................................... 920
Income before income tax.............................................................................. 132,875
Income tax expense....................................................................................... 27,904
Net income..................................................................................................... $104,941

*198. The cost of goods sold section of London Industries Ltd.’s income statement for the year ended
December 31, 2018 is shown below. For each missing item, choose the item number from the list below to
complete the report.

London Industries Ltd.


Partial Income Statement
Year Ended December 31, 2018
Cost of goods sold      
(a) __________________________________   $ 78,500  
(b).__________________________________ $185,400    
Less: (c) _____________________________ 4,250    

Copyright © 2017 John Wiley & Sons Canada, Ltd. Unauthorized copying, distribution, or transmission of this page is prohibited
Test
5 -Bank
70 for Financial Accounting: Tools for Business Decision-Making, Seventh Canadian Edition

Purchase discounts 2,800    


Net purchases 178,350    
(d) __________________________________ 6,580    
(e).__________________________________   184,930  
(f) __________________________________   263,430  
(g)._________________________________   52,700  
Cost of goods sold     $ 210,730

*Solution 198
(a) Inventory January 1 7
(b) Purchases 4
(c) Purchase returns and allowances 3
(d) Freight in 5
(e) Cost of goods purchased 6
(f) Cost of goods available for sale 1
(g Inventory Dec. 31 2

The following is the completed cost of goods sold section for London Industries Ltd.’s income statement:

London Industries Ltd.


Partial Income Statement
Year Ended December 31, 2018
Cost of goods sold
7. Inventory, January 1 $78,500
4. Purchases $185,400
Less: 3. Purchase returns and allowances 4,250
Purchase discounts 2,800
Net purchases 178,350
5. Add: Freight in 6,580
6. Cost of goods purchased 184,930
1. Cost of goods available for sale 263,430
2. Inventory, December 31 52,700
Cost of goods sold $210,730

*199. The following is a selection of journal entries that Giraffe Company Ltd. recorded for the month of August
2018. Giraffe uses a periodic inventory system. Indicate all of the entries that were correctly recorded.

(a) Aug 1 Purchases 72,000


Accounts Payable 72,000
(To record goods purchased on account from Llama
Company, invoice #L134, terms 2/15, n/30, shipped FOB
Copyright © 2017 John Wiley & Sons Canada, Ltd. Unauthorized copying, distribution, or transmission of this page is prohibited
5 - 71 Merchandising Operations

shipping point)

(b) Aug 1 Purchases 3,785


Cash 3,875
(To record payment of freight on goods purchased from
Llama Company)

(c) Aug 5 Accounts Payable 17,500


Inventory 17,500
(To record return of damaged goods purchased on
account from Llama Company)

(d) Aug 14 Accounts Payable 54,500


Cash 53,410
Purchase Discount 1,090
(To record payment of accounts payable to Llama
Company)

(e) Aug 15 Accounts Payable 18,500


Cash 18,500
(To record payment for goods purchased from Hippo
Industries in July – no discount taken)

(f) Aug 16 Accounts Receivable 115,000


Sales 115,000
(To record goods sold to Zoo Inc., Invoice 13425, Terms
2/10, n/30)

(g) Aug 16 Freight Out 5,890


Cash 5,890
(To record freight costs on sales to customer shipped
FOB shipping point to Zoo Inc.)

(h) Aug 24 Cash 112,700


Accounts Receivable 112,700
(To record receipt of payment on account for sales to Zoo
Inc., Invoice 13425)

*Solution 199
(a), (d), (e), and (f) are correct; (b), (c), (g), and (h) are not correct.

(b) Under a periodic inventory system, freight costs are recorded to the freight in account, as follows:
Freight In 3,785
Cash 3,785

(c) Under a periodic system, purchase returns and allowances are kept track of by recording returns to the
purchase returns and allowances account, as follows:

Accounts Payable 17,500


Copyright © 2017 John Wiley & Sons Canada, Ltd. Unauthorized copying, distribution, or transmission of this page is prohibited
Test
5 -Bank
72 for Financial Accounting: Tools for Business Decision-Making, Seventh Canadian Edition

Purchase Returns and Allowances 17,500

(g) Giraffe shipped the goods FOB shipping point, the shipping costs are the responsibility of the customer,
not Giraffe. Therefore, this entry should not be made and the shipping costs should be paid by the
customer.

(h) The company did not record the discount taken by Zoo Inc. The entry should be:
Cash 112,700
Sales Discounts 2,300
Accounts Receivable 115,000

Copyright © 2017 John Wiley & Sons Canada, Ltd. Unauthorized copying, distribution, or transmission of this page is prohibited
5 - 73 Merchandising Operations

LEGAL NOTICE

Copyright © 2017 by John Wiley & Sons Canada, Ltd. or related companies. All rights reserved.

The data contained in these files are protected by copyright. This manual is furnished under licence
and may be used only in accordance with the terms of such licence.

The material provided herein may not be downloaded, reproduced, stored in a retrieval system,
modified, made available on a network, used to create derivative works, or transmitted in any form or
by any means, electronic, mechanical, photocopying, recording, scanning, or otherwise without the
prior written permission of John Wiley & Sons Canada, Ltd.

Copyright © 2017 John Wiley & Sons Canada, Ltd. Unauthorized copying, distribution, or transmission of this page is prohibited

You might also like